Вы находитесь на странице: 1из 222

SLE Collection work 2011-2012 (from August to December 2011)

First "Updated" Edition 29-12-2011 collected by : -Qassim College of medicine : -King Saud University : and others - Umm Al-Qura University : and others King Khaled University
note : sorry ,there are some questions without answers ( as some of them not sure about correct answer and others have mention in 2010 Qassim collection ) in next editions we will try to answer all questions inshallh -organization and Supervision : Abdullah Saleh AlHudaib -Head of MD26 Interns Physicians groups, Qassim University MD26IPG@gmail.com in cooperation with : Study SLE Together Group on face book

SLE OROMETRIC 26 /9/2011 DR. AHMAD A ALSHOMAR,Qassim College of Medicine Please dont forget to prey to me ) the most common cause of nipple discharge in non lactating women is ; a-prolactenoma ( my answer ) b-hypothyroidism c- breast CA

d-fibrocystic disease with ductal ectesia .

please check the correct answer because I m not sure 2-seniro with patient has fear , SOB , sweating when he is in automobile DX

b-panic disorder

a-specific phobia ( the correct answer ) c-generalize anxiety disorder

d- post traumatic stress disorder

heavy bleeding 3 dysfunctional uterine bleeding

How to manage ? a-combined oral contraceptive pill ( the correct answer b-hospitalize and give blood transfusion

c-hysterectomy 4- polymyalgia Rhematica case with elevated ESR , other feature ; d-do D& a-proximal muscle weakness b-proximal muscle tenderness 100% (correct) cd5- senior female patient with hiatal hernia ; which of the following correct ?? a-it become more severe in pregnancy ( my answer ) bc6- regarding barret easophgitis which correct ? b-risk of squmou cell CA ca-risk of adenocarcenoma 100% true

7- man change his job , he must in new job to talk in front of 50 persons , he feels that he can not do this and he send his friend to do that instead of him, who can you help him ? 1. propranolol 2. Biofeedback ( my answer ) true 8- pregnant LADY GIVING HISTORY OF INCREASED BODY WEIGHT ABOUT 3 KG FROM THE LAST VISIT AND LOWER LIMB EDEMA TO CONFERM THAT SHE HAD PREECLAMPSIA what u want to cheek : a-measure BP ( my answer ) b-protein urea c-edema

9- pregnant lady with hyperthyroidism what you will give her : propylthiuouracil (my answer ) methamazole B blocker Radioactive iodine

10-which of the following correct about positive predictive value : 11- Which heart condition is tolerable during pregnancy:???

a. Eisenmenger syndrome

b. Aortic stenosis ( my answer )

c. Severe mitral regureg ( I am not sure bcoz the regure can be tolerable in pregnancy but in case of sever regure I dont know ) d. Dilated cardiomyopathy with EF 20% my answer is b but I think the correct is c please cheek it a. Antibiotics b. No ttt c. e. Mitral stenosis and the mitral area is 1 cm (or mm).

12-- 35 year old smoker , on examination shown white patch on leucoplakia the tongue, management: Close observation

d- excision biopsy ( may answer )

13-6 yrs old child came to you he only had his BCG vaccine, HbsAg +ve (mother also +ve) wt to give: -DTP,OPV,HiB,HepB,MMR -.......... -DTP,OPV,HiB,MMR ( my answer )

14- what is the causative organism of infectious mononucleosis ?

a-EBV ( true)

15- male singer with colon cancer stage B2 ; which of the following correct ? a- no lymph node metastases c-2 === b-one lymph node metastasis the correct answer is a

d-lymph node metastasis + distant metastasis Stage IIB T4a, N0, M0: The cancer has grown through the wall of the colon or rectum but has not grown into other nearby tissues or organs (T4a). It has not yet spread to the nearby lymph nodes or distant sites

16- child swallowing battery in the esophagus management : -bronchoscope( correct answer) - insert fly catheter - observation 12hrs ( my answer) -Remove by endoscope

17- wound at end inflammatory phase which of the following correct : Epithelial tissue formation

-angiogenesis is complete

Wound clear( my answer)

Wound eschar formation-Correct answer18- Greatest reversible risk of stroke: DM Family history of stroke Hyperlipedemia Smoking Elevated blood pressure ( my answer )

18- colon cancer with stage 3 give the chemotherapy: As soon as possible ( my answer ) After psychological prepare -After 1 week 19- TTT of hallucination and delusion ? a-antipsychotic ( true ) b-

19- An elderly lady presented with chronic knee pain bilaterally that increases with activity & decreases with rest. The most likely diagnosis is:

a) Osteoarthritis ( true ) b) Rheumatoid arthritis c) Septic arthritis d) ..

20-A pregnant lady, 8 weeks gestation, came with Hx of bleeding for the last 12 hours with lower abdominal pain & she passed tissue. O/E the internal os was 1cm dilated. The diagnosis is: a) Complete abortion c) Missed abortion d) Molar pregnancy b) Incomplete abortion ( true ) e) Threatened abortion

21- A 34 year old lady presented with pelvic pain and menorrhagia. There is history of infertility. On examinations the uterus was of normal size & retroverted. She had multiple small tender nodules palpable in the uterosacral ligament. The most likely diagnsosis is: a) Fibroid d) PID b) Endometriosis ( true ) c) Adenomyosis

22- similar case about endometriosis ; the beast way to investigate ?

a-US

b-repeated BHCG d- hysteroscopy

c-laparoscopy (my answer ) 23- typical senior of acute cholycyctitis ; the beast way to investigate ? a-US ( my answer) b-x-ry c-

24- Treatment of herpes zoster in ophthalmic division:or how to prevent post herpitic neuralgia ; a) Oral acyclovir alone c) Prednisolone b) Acyclovir & Prednisolone true 100%

25- Lichen planus most common site ? Scalp Knee ( my answer) Buttocks

d) IV Acyclovir

26- Relation of indirect hernia to spermatic cord Superior medial Inferior medial

Mouth ???

Inferior lateral

Superior lateral (like anteriolateral) ( my answer )

27- Old pt complaining of back pain on walking on examination there was stiffness of the muscle and there was some finding on the X-Ray best effective ttt Physiotherapy ( my answer ) NSAID Surgery

28- Pt came with deep injury on the wrist site, the nerve that has high risk to be injured will manifest as? Can not oppose thumb to the other finger ( my answer ) Claw hand Drop hand

29- Pt work most of the time on the computer came with wrist pain , positive tinel sign you will do cast for the hand so the hand position should be in

A-Dorsxiflexion ( I think its correct answer A or D b-Planter flexion C-Ulnar deviation

D-Extension ( my answer )

30- Patient after accident, there was a part on his left chest moving inward during inspiration and outward during expiration Dx Pneumothorax Rib fx Flail chest ( my answer ) Rib dislocation

31- old male bedridden with ulcer in his buttock 2 *3 cm ; involve muscle Which is stage : pressure ulcer a-1 c-3 b-2

d- 4 correct answer

stage I : non-blanchable redness that NOT subside after relive of the pressure) -stage II : damage to epidermis & dermis but NOT deeper

) -stageIII : subcutaneous tissue involvement)

-stageIV : deeper than subcutaneous tissue as muscles & bones)


st

32management ; b-give O2 cd-

a-maintains airway ( may answer )

step in

33- young male patient present to ER due to RTA with poly trauma ; the beast way to maintains airway in responsive poly trauma patient is ; A-orophargenial airway c-trachastomy b-nasophargenial airway

d-endotracheacheal intubations ( my answer )

34- long case patient with RTA with Blount trauma to abdomen . patient undergo remove of distal small intestine and proximal colon , patient come after 6 month with chronic diarrhea , SOB , sign of anemia , CBC show megaloblastic anemia What the cause of anemia :

A-folic acid deficiency bc-alcohole d-

35-male patient present with swollen erythem , tender of lt knee and rt wrist , patient give history of international travel before 2 month , aspiration of joint ravel , gram ve diplococcic , what is most likely organism ; a-nesseria gonorrhea (my answer) b-staph coccus dc-strepto coccus

36)the initial non pharmacolgical measurment in osteoartharitis is : a-steringth of qaudrcepc muscle ( the correct answer ) bc-

37- the beast exersice that increase muscle density and strengths of bone mass is : ????

38-lang senior , patient with Grenache nasal discharge , sinus pressure last 4 month , He ttt with broad spectrum antibiotics with no response , a-antihistamine d-antibiotic b-local decongestion e- observation

chronic sinusitis not response to antibotuc , what is the management now ; c-local corticosteroid ( the correct answer )

Young patient with congested nose, sinus pressure, tenderness and green nasal discharge, has been treated three times with broad spectrum antibiotics previously, what is your action? a) Give antibiotic b) Nasal corticosteroid c) Give anti histamine The correct answer is b d) Decongestant

39- child pt. came with scenario of chest infection , first day of admission he treated with cefotaxime , next day , pt state became bad with decrease perfusion and x-ray show complete rt. Side opcifaction + hydrothorax , causative organism :

Strepto. Pnem( my answer ) Pseudomonas

Staph. Aureus true if pnumothorax Hemophilus influenza type b A-candida albicans 40- case infant has genital rash ( the rash spares genital fold ) not response to antibiotics , most likely Dx; b-napkin dermitis ( my answer ) c-contact dermatitis d- atobic dermatitis e- sebborich dermatitis 41- 13 years old child with typical history of nephritic syndrome ( present with an urea , cola color urine , edema , HTN ) what is the next step to DX . a-renal function test ( my answer ) b-urine sediments microscope c-US d-renal biobsy

42-long senior patient came with chest pain , burning in character , retrsternal , increase when lying down , increase after eating hot food , clinical examination normal DX GERD a-MI

b-peptic ulcer d-

c-GERD ( correct answer )

43-lady with - Breast feeding 10 month child ,the mother has convulsion ,,,he take phenoparbital : phenoparbital

Weaning withen 3 weeks - stop - do not stop - after 8 hours

Lactating mother newly diagnosed with epilepsy , taking for it phenobarbital you advice is: a. Discontinue breastfeeding immediately b. Breas@eed baby aAer 8 hours of the medica_on the correct answer is c c . Continoue breastfeeding as tolerated -very vague question , some books avoid Phenobarbital during breast feeding if possible. And in American academy of pediatric classified Phenobarbital as adrug nursing women with cation . that cause major adverse effect in some nursing infant, and should be given to

44-lady with big abscess in left arm , how to manage ; a-antibiotic cdb-antibiotics and incion & dringe ( my answer)

45- female about 30y c/o abdominal pain related to menses (scenario going with endometriosis)next step in dx: Laparoscopy my answer U/S CT 46- about head and neck injury : Hoarsness of voice and stridor can occure with mild facial injury ( my answer) Tracheostomies contraindicated Facial injury may cause upper air way injures

47- about fetal alcohol synderome ??

48-female pregnant has HIV +ve , what is the most accurate information to tell her about risk of transmition to baby ;

A-likely transmtion through placenta b-through blood cord d-by breast feeding c-hand contamion of mother

most likely correct answer is b

49- the beast way to ttt pinged induce nervosa ( bullima nervosa ) a-interpersonal psychotherapy c-pharmacotherapy db-cognitive behavior therapy ( true )

50- 35 year old smoker , on examination shown white patch on the tongue, management: a. Antibiotics b. No ttt c. Close observation d. ---------

-This is a case of leukoplakia and the management includes:ask the pt. to stop smoking, do a biopsy for the lesion; if there is precancerous changes or cancer in the biopsy ; surgical excision should be done.

51- about vareciall vaccine in adult , which is true ; a-2 vacceine abart of 1 month b-2vaccine abart of 6 month d-3 vaccine abart of 6 month c-2 vacceine abart of 2 month the correct answer is Two doses are always recommended. In the first dose at age 4-6 years. For people older than 13 the two doses are administered 4 to 8 weeks apart.[17] 52- there is outbreak of diphtheria and tetanus in community , regarding to pregnant woman: a. contraindication to give DT vaccine b. if exposed , terminate pregnancy immediately c. if exposed , terminate after 72 hour d. give DT vaccine anyway -The correct answer is d. d. give DT vaccine anyway

30 year woman with dysmenorrhea, menorrhagea, infertility, and on examination found immobile mass on uterosacral ligaments : a. uterine fibroid b. endometriosis c. ---

-The correct answer is b 53- Most common symptoms or sign of renal cell carcinoma in adult is a. Hematuria b. Abdominal mass c. Flank pain -The correct answer is a. d.

54- pt taking bupropion to quit smoking what is SE a. Arrythmia b. Seizure c. xerostomia The correct answer is b d. Headache??

b+ d

55- your advice to prevent plaque disease is

a-hand washing b-rodent eradication ( ???) 56- pregnant with insulin dependant with good control, so to decrease risk of congenital disease c-spry insect side a-good metabolic control before pregnancy b-"""""""""""""""""""""""1st trimester c-""""""""""""""""""""""""2nd """""

d-""""""""""""""""""""""""3rd """"""" a-lithium ( true 100%) b-Na volabrate cd-

the correct answer is a (I'm not sure )

57-the drug used in maintance phase of biopolar is :

58- 6 years child was born to HBS positive mother is HBS positive , he was only vaccinated by BCG after birth , what you will give him now : a. HBV + oral polio + DTP + hib c. oral polio + Dtp + MMR+ hib ( true ) b. HBV + oral polio + dt + MMR +hib

59- The useful excurcise for osteoarthritis in old age to maintain muscle and bone a- Low resistance and high repetion weight training:

a. Conditioning and low repetion weight training the correct answer is b b. Walking and weight exercise

Exercise is one of the best treatments for osteoarthritis. Ask your doctor for an exercise prescription based upon your particular situations. The best

exercises for osteoarthritis suffers depend on what joints are affected. Swimming, walking, and cycling are often the best exercises for people with osteoarthritis. Try to get thirty minutes of exercise five times per week. The key is to start slowly.

60-old male with stroke , after 9 day he loss left eye vision , what are the affect structure ; a-frontal lobe b-partial d-temporal c-occipital ( my answer ) 61- a-somatization b-malingerinc ( true )

c-depression

62- The best way to reduce the weight in children is: a. stop fat intake b. Decrease calories intake d- decrease CHO c. Drink a lot of water e- multifactorial intervention with family ( my answer) 63- Patient came to you and you suspect pre eclampisa, which of the following will make it most likely: a. Elevated blood pressure b. Decrease fetal movement The correct answer is a c. ?? a-blood dissemination c- joint invasion d-

64- regarding group A streptococcus infection , how lead to rheumatic fever; b-by causing pharngitis /tonsillitis ( my answer )

65- the most common regimen in TTT of uncomplicated community acquired pneumonia ; a-azithromycine ( my answer)

b-flouroqunlone c-penicilline d-gentmycine

67- A boy felt down on his elbow , the lateral x-ray shows: a. Anterior Pad sign b. Posterior pad sign (my answer ) c. Anterior line of humerous intersecting the cubilium 68- Which of the following medication if taken need to take the patient immediately to d. Radial line forming 90 degree with cubilium the hospital: a. Penicillin b. diphenhydramine d. Quinine or Quinidine ( my answer ) c. OCPs How to manage ; a-CS ( my answer ) b-spontius delvery c-forceps delivery d- do amniotomy

69- case 38 wks pregnant lady with placent brevia marginal with mild bleeding , cervix 2 cm

70- Infant born with hemangioma on the right eyelid what is appropriate time to operate to prevent amylopia: a. 1 day b. 1 week ( 1000% correct answer true ) c. 3 months difficult Q. d. 9 months 71- Patient was presented by ear pain , red tympanic membrane , apparent vessels ,

with limited mobility of the tympanic membrane , what the most likely diagnosis : A. Acute otitis media . B. Tympanic cellulitis . The correct answer is a C. Mastoditis .

72- Benign tumors of stomach represent almost : A. 7 % ( my answer ) B. 21 % C. 50 % D. 90 %

73- Patient was presented by back pain relieved by ambulation , what is the best initial treatment : A. Steroid injection in the back . B. Back bracing . The correct answer is c C. Physical therapy . 73- Child came to ophthalmology clinic did cover test, during eye cover , his left eye move spontaneously to left, the most complication is: a) Strabismus b) Glaucoma c) Myobloma The correct answer is a 100% d) ?

74- The most common active form of thyroid hormone is: a) T4 b) T3 ( my answer) c) TSH d) TRH e-T2 the active form is T3 but the highest level is T4

73- y old patient , farmer , coming complaining of dry eye , he is smoker for 20 years and smokes 2 packs/ day , your recommending : a. advise him to exercise b. stop smoking The correct answer is b , smoking increase risk of dry eye (international study c. wear sunscreen 74- most important point to predict a prognosis of SLE patient : ?? a. degree of renal involvement b. sex of the patient the correct answer is a c. leucocyte count 75- what is the prophlaxis of mengiococcus meningitis a- rifimbcine\

76-how to prevent malaria 77- male old patient has S&S of facial palsy ( LMNL) ; which of the following correct about it ; A- almost most of the cases start to improve in 2ed weeks b- it need ttt by antibiotic and anti viral

c- contraindicated to give corticosteroid d- usually about 25 % of the cases has permanent affection correct answer is A

Please dont forget to prey to me

Q) in cachectic patient, the body utilize the proteins of the muscles >> to provide Amino acid and protein synthesis Q)-Thyroid cancer associated with: >> Euothyroid / hyper / hypo /graves Q) patient with recurrent pneumonia and productive cough , foul smelling sputum increase with lying down + clubbing >>> bronchectasis / BA / pneumonia Q) long constibation + painful defecation persist for 30 min + bleeding >>> anal fissure Q )pt has HTN come with pulstile abdomen swelling >> aortic aneurysm / renal cause / etc.. Mechanical IO / ileus / ets Q) Facial nerve when it exits the tempromandibular joint and enter parotid gland it passes: >> Superficial to retromandibular vein and ext. carotid artery / deep to ex. Carotid / deep to R vein

Q) pt with vomiting , constipation ,pain and distension past hx 7 month appendectomy dx ;

Q) flu like sx since to days and now has red eye ( pic ) Dx:

Viral conjunctivitis / bacterial conjunctivitis / uvitis / glaucoma Q) according to hemorrhoid >> can be due to portal HTN & pregnancy Mitral stenosis / CHF / endocarditis

Q) young pt came to ER with dyspnia and productive tinged blood frothy sputum , he is known case of rheumatic heart dz , AF and his cheeks has dusky rash dx : Q) Malaria : the most common cases is caused by Plasmodium falciparum. Q) 5 y.o child with h.o fever and swelling of the face ant to the both ears (parotid gland enlargement) what is the most common complication >>meningitis / labrynthitis / orchitis Q) self breast examination >> monthly First 10 kg X 100ml >> 1000 ml Second 10 kg X 50ml >> 500ml Third 5 kg X 20 ml >> 100 ml Total = 1600 ml Q) You r supposed to keep a child NPO he's 25 kgs, how much you will give for maintenance >> 1600 ml .

Q) girl with band like headache increase with stress and periorbital , twice / week >> tension headache / migrin / cluster

Q) old pt take hypercalcemic drugs and developed gout what is responsible drugs >> frosamide / thiazide Q)lactating women with mastitis >> continuo breastfeeding

Q) lady drive a car and can't see the traffic light ( which one test the distance ) >> snelln chart / tonometer

Q) 56 y old present with vasomotor rhinitis >> Local anti histamine /Local decongestion /Local steroid / Systemic antibiotic . Tonic-clonic seizure activity lasting > 5 to 10 min 2 seizures between which patients do not fully regain consciousness

Q) status epileptics >> Continuous sizure activity more than 30 min without regaining consciousness ??? the choices wsa confusing BUT the new definition ;

Q) In pt with moderately sever acne valgarus best ttt >> Oral isotretinoin / topical Retinoids /Topical clindamycin / oral antibiotics

Q) which of the following TTT contraindication in asthmatic pt >> Non-selective B blocker What is most appropriate inx to DX gall stone ? US / Xray / Barium

Q) case with 60 years old male with RT upper quadrant pain after dinner , most likely DX gallstone ;

Q) another case , typical case acute cholcystits , What is most appropriate ivex to DX colycystits : Abdominal ultrasound / oral cholycystogram / isotope scan

Q) long case , patient fall down from ladder , come to ER with labored breath , cynose , decrease breath sound on rt side + hyper resonse , management is ? O2 via mask / tube throctomy / endotrocheal tubation Q) long case , acute pancreatitis which is TRUE; NB; there was NO needle decompression or chest tube in choices

Total parental nutrition / Regular diet with low sugar /High protein ,high ca , low sugar / Naso-jujenal tube Q) human bite to hand , most common hand position that proposed to infection ? Clenched hand / dependent / extended thump / extended fingers Q) TTT of opiod toxiacity >> Naloxin Q) medication induce ovulation >> clomphine citrate

Q) In chlamedia infection ttt is ? ( not mention pregnant or not ) Cervix eroded + friable DX ;

Doxcycline / Azithromycine / Metroniadizole ( also, doxcy used ) Trachimonus vaginits / Chlamydia / Nesisseri gonnerhia Congenital CMV

Q) Cervictitis + strawberry cervix + mucopurelnt yellow disharge

Q) case infant , hepatospleanmgly , , jaundice , what is the dx ? Q) most common cause of renal failure >>DM Q) strongesrt factore for intracerebrah hemorrhage >> HTN

Q)case cord like cheesy white adherent odour less vagina after use of antibiotic DX >>Candidiasis Q) case of nesseria gonnerha , the beast TTT ? Peniciline G / Ceftrixone (( make sure plz ))

Q) Relation of indirect hernia >> Antero lateral or supralateral

Q) PTs complaint of loss of association and cirumstantionciality the defect in >>>> form NB; FORM ; loss of association , circumstanciality , neologism and flight of idea CONTENT ; delusion , obsession and phobias . Q)- malaria case , beside antiobtic how to prevent ? Kill the vectore and ..

Q) which of the following increase bone density and muscle strength >> Andurance and whigh exercise / high repetition . / low repetition . Q)Positive predicitive value : Definition ? Answer was >> C " pt who has high Risk factor & +ev test "

Q) Pt with HTN and multiple risk factors " obese +high sodium intake +alcohol intake + high potassium " which is most important RF for HTN ? >> Obesity / High Na intake/ High K intake / alcohol

And the most important action >> wt reduction

Q) Femal come to family physician ask about diet that decrease CVD , ( She has family hx) ? Q) Most effective method for health education ?

Increase fruit and vegetable / Decrease the intake of meat and dairy / Decrease the meat and bread . Mass media / Group discussion / Internal talk (true) Person to person / Vector / Droplet /Air flow

Q) Most difficult method to prevented in transmission:

Q) Child with morbid obesity , what the best advice for him: Q) The most active for of thyroid function test >> T3

Decrease calories intake / Dec fat intake /Increase fiber / incr. water

Q) Case of hemangioma in the eye affecting vision , when you have to remove >> 1 week sure 100% inshallah Q) Clear case of osteoarthritis ( bilateral knee pain incre with activity . Physiotherapy ( because its mild ) / NASID / surgery

Q) Case back pain on walking and stiffness on muscle radiology show spinal stenosis , best ttt ?

Q) In diabetic retinopathy , most related factors:

HTN and obesity (sure ) / HTN and smoking / Smoking and obesity

Q) Scoliosis, when to refer the patient to surgery >> 20 degree ( sure )

Q) Patient is known case of cervical spondylolysis , presented by parasthesis of the little finger , with atrophy of the hypothenar muscles , EMG showed cubital tunnel compression of the ulnar nerve , what is your action now?

Ulnar nerve decompression / Steroid injection / CT scan of the spine Q) Newborn came with red-lump on left shoulder, it is >> Cavernous Hemangioma ( my answer) Streptococcal gangrene / Chlostrideal gangrene / Fourniers gangrene / meningocemia Syphilis / HSV /Chancroid Q) 20 year old male had been stabbed on midtriceps , one week later greenish discharge , On microscopic examination of this greenish fluid show gram positive cocci in chain ? Q) female young with dew tear vesicles on rose red base and painful on valve ? Q) 9 yrs pt come with ear pain , red tense tympanic membrane , -ve Rhine's test with + ve Weber test with lateralization ( conductive loss) for TOW days only ? Otitis media / otosclerosis / cholestiatoma

Q) The same case above BUT he said conductive hearing loss directly without those tests >> Otitis media Q) pt was PDD ve , know become + ve , there is no symptoms , normal x ray, the management : Q) pt with COPD, Which of one increase surveillance ? O2 home therapy /Steroid / ibratropium Reassure / Rifambicin and INH for 6 month / Streptomycine for 7 month / rifambicin for 6 months .

Q) pt come to ER with AF, BP 80/60 what it the management: synchronized CD / Digoxin UTI / GIT

Q) old pt, bedridden , with bactermia , organism is enterococcus fecalis , what the source of infection: Q) 4y girl, decrse head growth, decrse social intraction, decrase in language etc: Rett's syndrome Beta blocker( there is no ssri in choices)

Q) youg female become flushing face and tremors when she talk to any one what ttt: Q) pt a afraid to go outside >> agoraphobia Q) case of Raynaud's phenomenon it was direct >> pallor then cyanotic then red finger without other clinical features . Q) read about rebound hyperglycemia in DM ?? somogi and down phenomenon .

My exam done in 18 October 2011 Hope this collection help you in your exam Very spical thanks for dr. Ahmad Alshomar and dr.hussam alsulmi because the help me in remembering the Qs and in writing Dr. Yasir Abdulmohsen Alrusayni,Qassim Co0llege of Medicine Wishing for you all the best

SLE-15/10/2012 ) . 1. PTS come with history infertility complaining of decrease period , acne,hirstusim diagnosis is >>>>>> PCOD 2. PTS with history of infertility the first line of investigation for this couple is >>> semen analysis 3. PTS come with history of infertility of 6 months unprotective intercourse >>>you must complete 12 months of unprotective intercourse 4. Old PTS with depression and you prescribed SSRIs for him < counsel for the PTS is >>>>> take 3-4 wks to produce action 5. PTS with depression manifestations , what is the mechanism of the drug that you will prescribe >>>> increase availability of serotonin 6. Why the SSRIs is the first line of treatment of depression >>> effective and tolerable 7. PTs inside his home catch and cover the TV , and when ask him why to do this , he said the government follow him by watching and listening to his actions, he said the God told him about this diagnosis >>>>>Schizophrenia

8. PTs complaint of loss of association and cirumstantionality the defect in >>>> form 9. libido , loss of concentration , wt gain since hot flush , affect marital state >>>>> estrogen???? / progesterone / fluxatine ?????? 10. adult PTS with history of anemia sickle cell , he at risk of >>> infarction 11. Child with history of SCA and recently treated from acute crisis before discharge what you will give him >>>>> penicillin ?????? 12. Old PTS with history of recent MI complain of pain of RT leg , on examination absence of pedal puls , cold RT leg and normal LT leg diagnosis is >>>acute Arterial embolism . 13. Old PTS with history of recent MI complaining of sever abdominal pain , distention , bloody diarrhea, slightly raised serum amylase diagnosis is >>>> Ischemic colitis ??? 14. Old pts with history of bilateral pain and crepitation of both knee for years now come with acute RT knee swelling , on examination you find that there is edema over dorsum and tibia of RT leg ,what is the best investigation for this condition >>>>Rt limb venogram 15. An elderly lady presented with chronic knee pain bilaterally that increases with activity & decreases with rest. The most likely diagnosis is<<< osteoarthritis 16. What is the definition of epidemical curve >>>graphic registration of disease through a period of time ????? 17. Standard deviations >> measurement of central tendency. 18. PTS 18 yrs , you prescribe for him retinoid gel will counsel him for >>>>> make your skin sensitive for sun light ????? 19. In a flame burn what is the cause of acute death >>> Gas inhlation 20. Sliver ??? ???? drug used in Burn , what is the side effect >>> leuckopenia???? 21. Conscious poly trauma pts , what is the action >>> ABC

22. Pts hit on his chest , after 2 hours come with , BP 100 /70 , pulse 120 , RR 40 , chest x-ray show, white lung field in the LT hemithorax , what is your action >>>> thoracoectomy. 23. What is the name of questionnaire that differentiate b/w primary and sleep apnea.??????? 24. Old pts with positive occult blood in stool >>>> flexibale sigmoid scopy? / colonscopy 25. Young pts come with sever testicular pain , decrease in doplex supply to tests, what is your action >>>> refer to surgen / refer to urologist / more investigation 26. Female with dysuria on examination there is epithelial cell >>>> chlymdia urethritis 27. Post partum female with recurrent attack of hearing loss , which diagnosed as conductive hearing loss , on CT the is dehesion in the of semi circular canal diagnosis >>>> otosclerosis / miner's / Tuberus sclerosis . 28. When the spinal length stop after menarche >>>>> 1 yr / 2 yrs / 6 months ??? 29. Infant newly giving cow milk in 9 months old , closed posterior fontanel, open anterior fontanel with recurrent wheezing and cough , sputum examination reveal hemoptesis , x-ray show lung infiltration , what is your action >>>> diet free milk / corticosteroid / antibiotics ?????? 30. One months Infant brings by his mother complain of bilious vomiting , constipation , abdominal pain , diagnosis by >>>>> rectal biopsy 31. Child with posing head , bowing tibia ,,,, rickets ,,, what is the deficiency >>> vit D deficiency. 32. 6 yrs +ve hepatitis , no vaccination , only BCG >>> what you will give >>>>OPV, DTP,MMR,Hib. 33. Which of the following is contraindicated to breast feeding >>>> asymptomatic HIV / Hepatitis c. 34. 80 year old man complain of sever itching mainly in the wrist and b/w fingers , with excoriation mark linear and superimposed by secondary infection disturbing sleep, the pts >> Monilia ???? eczema ??? icythiosis ????

35. Strongest method to prevent the disease >>> immunization / change health behavior of PPls 36. Female take OCPs come with skin changes on the face >>>> lupus lipura !!!! melasma ????? 37. Live guard come to annual examination , no compliant , macular dicloration, painless over the face , thers is history for exposure unproductive to sun rays >>>>>Sqamous cell carcinoma 38. Strongest risk factor to ostoprocesis is >>>> smoking / age /decrease exceasise . 39. Community problem of multiple chlymedia infection in the eye , best prevention method is >>>> good water and good sanitation supply . 40. Purulent discharge from ear middle ear how to treat him>>>> systemic AB/local AB/steroid 41. Child with URTI then complained from ear pain on examination there is hyperemia of TM &+ve insufflations test

TTT>>>ugmentine/azythromycin / ciprofloxacin/steroid 42. Female pregnant 34w gestation complain from bleeding heaver than normal period O/E US show per placental lucency ,placenta implant normally post. In the fundus , uterine contraction every 4 minute,CX 3cm, fetal HR170 what is your action >>>>CS 43. Which IS considered abnormal & indicate fetal distress<<<late deceleration 44. What is the vector for leshmania disease <<<< sand fly 45. Infant in respiratory distress ,hypercapnia , acidosis & have rhinitis , persistent cough +ve aglutenation test & the doctor treat him by ribavirin DX>>>pertusus/RSV 46. Aluminum salt & salt will decrease absorption of <<<<tetracycline/penicillin/?????.. 47. 40 yrs old male com with HX of smoking & alcohol intake for long time complain of painless ulcer ,role out border on the lateral border of the tongue DX<<<<<SCC????/lukoplakia 48. Genital herpes CCC by >>>>> painful ,vesicular ,ulcer

49. 5yrs child have congested throat 2 day , complain of painless , clear , vginal secretion DX>>>>> foreign body 50. Old PTNs with osteoporosis TTT for HTN with diuretic that prevent Ca loss complain of severe pain in big toe DX>>>>>thizide 51. Old male come with CHF & pulmonary edema what is the best initial therapy>>>>digoxin/frosamide/debutamine 52. >>>>> family HX of early IHD/BMI>30/mother worry 53. Most effective way to stop smoking is >>>> PTNs desire 54. Painless penile ulcer what is next step>>>> dark filed microscopy 55. Patient with HX of URTI & flash of light when he sneeze the cause is>>>> chemical/ mechanical irritation of retena 56. Adult with HTN what is the most common cause >>>>> renal/essential 57. Adult with unilateral headache pulsetile increase with activity & light >>> migraine 58. What is the antiviral drug that cause fever ,chills &muscle pain <<< interfiron 59. All of the following exaggerate the gastric ulcer except<<<<decrease gastric empty time 60. Pregnant in 5 month gestation &on iron supplementation since that time &now com with dyspenia ,weakness &easy what is DX<<<<<thalasemia B/IDA 61. Adult only taken first dose of varcella vaccine >>>> give him second dose only 62. Carpal tunnel syndrome<<<<< dorseflexion 63. Patient is known case of cervical spondylolysis , presented by parasthesis of the little finger , with atrophy of the hypothenar muscles , EMG showed Ulnar tunnel compression of the ulnar nerve , what is your action now : A. cubital decompression . B. Steroid injection . C. CT scan of the spine

64. Adult with essential HTN BMI30-40,drink red win,high salt dite what is the best method to TTT HTN>>>>>>Decrease weight????/decrease win/decrease salte???? 65. Sever pain in anatomical snaph pox >>>>> scavoid fracture 66. Child come to ER after ingestion of multiple iron tablet of his relative &iron conc. In blood 700ml???? what is the best intervention>>>gastric lavage/charcoal oil/ iv defrroxamin 67. Patient presented with sore throat, anorexia, loss of appetite , on throat exam showed enlarged tonsils with petechi on palate and uvula , mild tenderness of spleen and liver :DX<<<<< infectious mononucleosis 68. Male patient have ARDS &on ventilation have manifestation of pnemothorax<<<-ve pressure ventilation/lung damage 69. Patient with lacremation ,salivation, diarrhea, what is antidote<<<<atropine&pralodexam 70. Child with recurrent UTI how to counsel him>>> increase fluid intake 71. Female com with lump in breast which one of the following make you leave him without appointment ( ) Cystic lesion with seruse fluid that not refill again?? Blood on aspiration Solid Fibrocystic change on histological examination ??? 72. What is the most common cause of nipple discharge in non lactating women prolactenoma hypothyroidism breast CA fibrocystic disease with ductal ectesia

intraductal papilloma

Old pt complaining of back pain on walking on examination there was stiffness of the muscle and there was some finding on the X-Ray best effective ttt>>>Physiotherapy

73. patient with DVT what is the cast effective TTT>>>>> LMWH 74. group of patient with lung cancer divided In tow group disease &control>>>case control study 75. DM1<<<<<< HLA DR4 76. Patient with contact lens 10 yrs complain of tearing on exposure to sun light<<<< eyes drops 4 daily 77. Alzheimer caused by <<<<< brain cell death?????? 78. Gouty arthritis -ve pirfringes crystal what is the mechanism>>>>deposition of uric acid crystal in synovial fluid due to over saturation 79. Wavy like abdominal contraction>>>> obstruction 80. Patient with injury to ant. Surface of wrist >>>>>unable to oppose the thump 81. senior female patient with hiatal hernia ; which of the following correct <<<<it become more severe in pregnancy 82. relation of indirect hernia to spermatic cord>>>>superiolateral 83. Patient have of urethritise now com with lt knee urethral swap +ve puss cell but ve for N.M, chlymedia>>>>RA/riters disease /gonococcal 84. Regarding group A strept pharyngitise what is true<<<early TTT decrease incidence of post strept GN 85. Pulled Elbow scenario the TTT>>>immediate reduction supination 86. Most active form of thyroid hormone is <<<<< T3 87. One of the following have anti cancer activity>>>> fiber / vitamin D 88. Child with asthma take steroid inhalation SE>>>>glaucoma /growth retardation

89.

50ml/Kg in 90. Which of the following indicate benign thyroid lesion>>>>lymphadenitis 91. BMI 24.5 >>>>> normal weight 92. Patient with pain full defecation ,fresh blood at the end ,sense of incomplete defecation>>>hemorrhoid 93. The best way to eliminate brucellosis is>>> milk pasteurization

Child with mild viral gastritis & you decide to give ORS

SLE OROMETRIC nov 2011

1- 35 yr old with painful eye movement and decrease visual acuity is having: a. Optic neuritis b. Retinintis pigmentosa c. Central retinal artery occulusion d. Central retinal vein occlusion 2- 80 yr old in his normal state of health presented with decrease visual acuity bilaterally without any defect in visual eld his VA Rt eye= 20/100 VA Lt eye=20/160 fundoscopic exam showed early signs of cataract and drusen with irregular pigmentations. No macular edema or neovasculirization. The appropriate action beside antioxidants and Zn is: a. Refer the pt for emergency laser therapy b. Refere the pt for cataract surgery c. See the patient next month d. No need to do anything 3- This pt has this painful lesion. The Dx:

a. Herpes zoster b. Folliculitis c. Cellulitis 4- Hypertensive patient on Thiazide presented at night with severe lt foot pain involving the first toe with redniss extending to the mid leg. The Dx: a. Cellulitis b. Septic arthritis c. Gouty arthritis 5constipation. He had previous abdominal surgery in the past. This is hix AXR: a. surgery for obstruction b. Rectal decompression c. Treatment of ilius

6- Pt with abdominal pain and distension with vomiting and constipation. He has mild symptoms of dehydration. There is evidence of air in the rectum. The Rx: a. Rectal decompression with IV antibiotics b. Nasogastric tube with IV isotonic fluid c. Systemic antibiotics 7- Patient with symptoms of blephritis and acne rosacea the best Rx is: a. Doxacyclin b. Erythromycin c. Cephtriaxone 8- Primigravida with whitish discharge the microscopic finding showed pseudohyphae the treatment is: a. Meconazole cream applied locally b. Tetracycline c. Metronidazole d. Cephtriaxone 9- Female with monilial vaginal discharge the treatment is: a. Meconazole cream for 7 days b. Fluconazole orally for one day c. Metronisazole orally for 7 days 10- A 56 yr old his CBC showed, Hb=11, MCV= 93 Ret= 0.25% the cause is: a. Chronic renal failure b. Liver disease c. Sickle cell anemia

d. G6P dehydrogenase deciency 11- 3 yr old his parents has a. Inconclusive result b. Weak positive result c. Strong positive result 12- An outbreak of TB as a prophylaxis you should give : a. Give BCG vaccine b. Rifampicine c. Tetracycline d. H. influenza vaccine 13- Pathological result from thyroid tissue showed papillary carcinoma, the next step: a. Surgical removal b. c. Give antithyroid drug d. Follow up the patient 14- Pathological result of cervical lymph node showed well differentiated thyroid tissue without any masses in the thyroid gland the best management is: a. Total thyroidectomy with radical resection b. Total thyroidectomy with modified resection c. lobectomy with radical resection d. lobectomy + isthmectomy with resection of the enlarged LN

15- which is true about gastric lavage: a. It is safer than ipecac if the patient is semiconscious b. It is done to the pt in Rt Decubitus position c. 16- 3 yr old ingested aspirin the best gastric decontamination is: a. Gastric lavage b. Activated charcoal c. Total bowel irrigation 17- The most dangerous condition in menopause is: a. Ovarian cancer b. Endometrial cancer c. Osteoporosis 18- Female pt with irregural menstrual cycle it comes every other month and lasts 7-8 days with a very heavy bleeding making her to put double pads yet these pads will be soaked completely. The best description is: a. Menorrhagia b. Polymenorrhea c. Metrorrhagia d. Metromenorrhagia 19- Null hypothesis : a. The effect is not attributed to chance

b. There is significant difference between the tested populations c. There is no significant difference between the tested populations 20- The specificity is: a. When the person does have the disease with +ve test b. When the person does have the disease with -ve test c. When the person does not have the disease with +ve test d. When the person does not have the disease with -ve test 21- The important risk factor for Stroke is: a. DM b. HTN c. Dyslipidemia 22- The mechanism of essential hypertension is due to: a. Formation of lipid outside the vessels b. Increase in peripheral vascular resistance c. Salt and water retention 23a. Respiratory alkalosis b. Hypoxia c. Hypothermia 24you would like to do: a. Renal function test b. Urine microscopic sedimentation c. Renal ultrasound 25- 65 yr old presented with acute hematuria with passage of clots and lt loin and scrotal pain . the Dx a. Prostitis b. Cystitis c. Testicular cancer d. Renal cancer 26- Which of the following radiosensitive: a. Choriocarcinoma b. Seminoma c. Yolk sac tumor d. Embryonic cell carcinoma 27and has female genital organs. The underlying process is: a. Prolactin over secretion b. Androgen over secretion c. 28would like to do in this acute state: a. Renal U/S b. Folly catheter

29-

30-

31-

32-

c. VSUG 30 yr old with repeated UTIs which of the following is a way to prevent her condition: a. Drink a lot of fluid b. Do daily exercise c. Pt with sudden cardiac arrest the ECG showed no electrical activities with oscillation of QRS with different shapes. The underlying process is: a. Atrial dysfunction b. Ventricular dysfunction c. Toxic ingestion d. Metabolic cause Which of the following is given as prophylactic ant arrhythmic after MI: a. Procainamide b. Lidocaine c. Qindine d. Quanin e. Metoprolol Pt with sudden Lt leg pain after attack of MI, the leg is cold and pale: a. Acute atherosclerosis of the leg b. Acute embolism of the leg c. Acute cellulitis dizziniss chest discomfort. She thinks about her jobe and children a lot . she is doing well at her job: a. Depression b. Panic attack c. Generalized anxiety disorder d. Social phobia Which personality disorder is associated with inflexibility and perfectionism: a. Narcissistic personality disorder b. Borderline personality disorder c. Obsessive compulsive personality disorder d. Histrionic personality disorder 5 yr old adopted child their recently parents brought him to you with white nasal discharge. He is known case of SCA. What you will do to him: a. Give prophylactic penicillin b. c. Which of the following antipsychotic associated with weight gain: a. Respiridone b. Quitapine c. Olanzipine d. Ziprasidone Which of the follwing antidepressant is not given in erectile dysfunction:

33-

34-

35-

36-

37-

38-

39-

40-

41-

42-

43-

44-

45-

a. Sertraline b. Amytriptaline c. Butriptyline d. Pt presented with nausea and vomiting and nystagmus with tinnitus and inability to walk unless he concentrate well on a target object. His Cerebellar function is intact: a. Benign positional vertigo b. meniere's disease c. vestibular neuritis 4 yr old with pain and swelling behind his left ear, he had symptoms of ear inflammation a week ago but he didnt took the medication. Now he has this pain with some inflammation inside the ear with loss of light reflex: a. Lt Otitis media b. Lt mastoiditis c. Lt Otitis externa 10 year old had an episode of rheumatic fever without any defect to the heart. The patient need to take the antibiotic prophylaxis for how long: a. 5 months b. 6 years c. 15 years Pt with difficulty getting air. Nasal exam showed unilateral swelling inside the nose. What is the initial treatment for this pt: a. Decongestant b. Sympathomimitics c. Corticosteroid d. Nasal decongestant can cause: a. Rhinitis sicca b. Rebound phenomena c. Nasal septal perforation The antibiotic prophylaxis for endocarditis is: a. 2 g amoxicillin before procedure 1 h b. c. 2 g clindamycine before procedure 1 h d. Child with fever and runny nose, conjunctivitis and cough then he developed Maculopapular rash started in his face and descend to involve the rest of the body: a. EBV b. Cocxaci virus c. Rubella virus d. Vaccini virus 19 yr old girl with URTI and splenomegaly. The cause: a. Infectious mononucleosis b. Streptococcus pharyngitis

c. Malaria 46- Child with leukemia he has septicemia from the venous line the organism is: a. E coli b. GBS c. Pseudomonas 47greenish showed Gram +ve cocci the organism is: a. Clostridium b. Streptococcus c. Pseudomonas

48- Pt after surgery the wound opened and secrete fluid this complication is: a. Colistridium infection b. Wound dehescience c. 49acute watarry diarrhea with stool leukocyte. The organism: a. C. perfernges b. C. difficele c. Salmonella 50any previous history, the stool sowed high fat he is a known somker: a. Acute pancreatitis b. Chronic pancreatitis c. Pancreatic carcinoma 51- When showed you refer a pt with scoliosis: a. 10 degree b. 15 degree c. 20 degree 52- What is the best way of health education: a. Mass media b. Internal talk c. Individual approach 53- The best way to prevent house mite: a. Cover the pillows with impermeable cover b. Wash the clothes in hot water c. Remove the old carpets 54- Pt with bilateral knee pain without signs of inflammation: a. RA b. OA c. Septic arthritis 55-139/80-85: a. Wight reduction and physical activity b. Exercise alone is not enough

c. 56- Pt with DM-II has conservative managenet still complaing of wight gain and polyuria a. Insulin short acting b. Metformin c. Long acting insulin

57- A father of twin want to ask you about the puberty of his children: a. Girls enter puberty 6-12 months before boys b. Girls enter puberty 2-3 years before boys c. Girls enter puberty 1d. Girls enter puberty as the same age of boys 58- The management of breast engorgement: a. Warm compression with continue breast feeding b. cold compression with stoppage of breast feeding c. cloxacillin with continue breast feeding 59BCG he should take: a. Td, Hib,MMR,OPV b. DTB,Hib,MMR,HBV,OPV c. DTB,Hib,MMR, OPV d. Td, Hib,MMR,OPV,HBV 60- Pt with good health presented with sudden shortness of breath with rt side hyperresonant the Dx: a. Spontaneous PTX b. Plural effusion c. Severe asthma d. Pulmonary embolism 61- 43 yr old sustained trauma to the chest present with severe short of breath with cyanosis, his rt lung is silent with hyperresonance. The FIRST step to treat this pt: a. O2 mask b. Tube thoracostomy c. CXR 62a. Needle decompression in the 2nd intercostal space midaxillary line b. Needle decompression in the 2nd intercostal space anterior-axillary line c. Needle decompression in the 5th intercostal space midaxillary line d. Needle decompression in the 5th intercostal space anterior-axillary line 63- Pt sustained a major trauma presented to ER the first thing to do: a. Open the air way give 2 breath b. Open the airway remove foreign bodies c. Give 2 breath followed by chest compression

d. Chest compression after feeling the pulse 64- Pt has pelvic inflammatory disease received ceftraixone but no improvement the propable organism is: a. Syphilis b. HSV c. Chlamydia 65didnt have any attack since. She want to stop taking the drug due to facial hair growth: a. It is reasonable to stop it now b. Stop it c. d. Dont stop it 66a. Cluster headache b. Migraine c. Tension headache 67- Which of the following is true about migraine: a. Aura occur after the headache b. c. It is unilateral pounding headache 68- Pt want to get pregnant so you gave her iron. Then presented to you with the following labs: Hb=9 MCV=60, the Dx is: a. Thalassemia b. Iron deficiency anemia c. Sidroblastic anemia 69- Pt with elbow trauma presented to yoy wich of the follwing is abnormal sign: a. Humeral head in continuation with the capitillum b. Posterior fad pad c. Anterior fat pad 70- Pt with 3 weeks history of shortness of breath with hemoptysis the appropriate investigation is: a. CXR,AFB,ABG b. CXR,PPD,AFB. c. CT,AFB,ABG 71- Pt with celiac sprue he should take: a. Carbohydrate free diet b. Protein free diet c. Gluten free diet 72- Pseudogout is Ca: a. Pyrophosphate b. Sulfate c. Uriate 73- First sign of MgSO4 overdose: a. Loss of deep tendon refelex

b. Flaccid paralysis c. Respiratory failure

74- A 43 yr old with bloody nipple discharge: a. Duct ectasia b. Duct papilloma c. Fibrocystic disease 75- Pregnant lady underwent U/S which showed anteriolateral placenta. Vaginal exam the examiners finger cant reach the placenta: a. Low lying placenta b. Placenta previa totalis c. Placenta previa marginalis d. Placenta previa partialis e. Normal placenta 76- Athlete who jogs on daily basis presented with groin rash with erythema, the Rx: a. Topical antibiotic b. Topical antifungal c. Topical steroid 77- Premature ventricular contraction is due to: a. Decrease O2 requirement by the heart b. Decrease blood supply to the heart c. Decrease O2 delivery to the heart 78- A mother brought her 9 days neonate with jaundice he is doing well with Icteric discoloration the cause is: a. Glucurenyl dehydrogenase deficiency b. Breast milk jaundice c. Fetal maternal blood transfusion 79- A pregnant lady full term presented with agitation and change level of consciousness she is having difficulty breathing: a. Pulmonary embolism b. Pulmonary edema c. Amniotic fluid embolism 80- Which of the following is a treatment for giardiasis: a. Prazequantil b. Mebendazole c. Metronidazole d. Albendazole 81- Which of the following is appropriate method to prevent brucellosis: a. Killing the vectors b. Prophylactic antibiotics c. pasteurization of the milk

82- pt after trauma to the knee present with knee swelling of bloody content , the probable mechanism is: a. platelet deficiency b. clotting factor deficiency c. platelet dysfunction d. blood vessels dysfunction 83- computer programmer presented with wrist pain and +ve tinnel test. The splint should be applies in: a. dorsiflexion position b. palmarflexion position c. extension position 84- proven effective method of improving the patients with osteoarthritis is by: a. walking and weight exercise b. c. 85- Treatment of community acquired pneumonia: a. Azithromycin b. Ciprofluxacine c. Gentamycine d. Tetracycline 86- Infant born with hemangiomas obstructing his vision to prevent Amblyopia the child need to undergo the surgery within: a. b. c. 87- Epidemiology is: a. b. c. 88- Pt with Rt arm tenderness with red streak line the axillary lymph node is palpable: a. Cellulitis b. Carcinoma c. Lymphangitis 89- Which is true about DM in KSA: a. Mostly are IDDM b. Most NIDDM are obese c. 90proteinuria the appropriate management is: a. Give oral labetalol b. Give MgSO4 c. Admit her till she deliver her baby 91- The most common cause of non traumatic subarachnoid hemorrhage is:

a. Middle meningeal artery hemorrhage b. Bridging vein hemorrhage c. Rupture of previously present aneurysm 92- Pt with DKA the pH=7.2 HCO3=5 K=3.4 the treatment: a. Insulin 10 U b. 2 L NS c. 2 L NS with insulin infusion 0.1 U/kg/hr 93- Treatment of severe depression whixh is resistant to treatment is by: a. TCA b. Electroencephalographic therapy c. Electroconvulsive therapy 94- A 45 yr old his lab showed Hb=11 MCV=93 a. Chronic renal failure b. Liver disease c. Thalassemia d. G6P dehydrogenase defeciency 95969798: \ \ \ \ \ \

My SLE exam

1. Compelete
a) Frontal b) Parital c) Cortex

Nov. 2011 loss of vision Lt eye, in pt with recent infarction? In the name of allah

2. Female with Hx of
d) Occipital

3. Most accurate test for CTS ? carpal tennel syndrome 4. Drug induced urticaria ?
a) Cemitidine b) Hydroxizia ?? a) Tinels test b) Phalens test c) Nerve tapping pumbness There was no nerve conduction velocity.

discharge ?? a) BID >>> mostly correct one b) Bacterial vaginosis Plz remember there was no ectopic pregnancy

recent IUD ,complaining of watery brownish

):

Penicillin , sulph , NSAID , Aspirin , antihistamine , codine , dextrose , rantidine , tetracycline , vaccine , etc Drugs induce urticaria are :

5. Hx of Child has itching in his RT hand

which increase in the night described as linear fissures at their top there is blacknish ? a) Scbias >> im not sure b)

6. Old male pt with diarrhea about 1 liter , how much he will loose 7. Which of the following method is rapid and best for complete 8. Polygonal skin lesion DX ??
gastric evacuation ? ...!!!! a) G lavage b) Manual induce V c) Syrupe d) Active charcoal from his weight? a) ,5 Kg b) 1 Kg > mostly c) 1,5 Kg

9. old male pt with hx of IHD, DM , HTN , dyslipidemia , 10.


family hx of heart disease ,,,, lab showed :- LDL : 199 ,,,, HDL : 37 so , in this pt what is most dangerous Risk factor ? a) Increased LDL >> correct one inshallah b) Decreased HDL Verrry loooong scenario , in short : Female has dysuria, lower abd pain , increase in frequency , voiding small amount of utine , HTN , ttted with multi ABx but no improvement . Lab : CBC , metabolic tests all are normal , wht is DX ?? a) Candidia albican b) Intrstital cystitis ???? c) DM

a) linchea planus remember it is pathognomonic .

11. 12. 13. 14.

:( ... ...

Old man psych pt , has halosination , aggressive bebaviour ,loss of memory ,Living without care , urinate on him self , what is next step to do for him ? a) Give antipsychotic b) Admit him at care center for elderly . mostly Pt with sever asthma , silent chest what is next step ? a) Iv theiophilen ??? my answer b) Neb salbtamol or sulmatrol >> is it work in silent chest I am not sure Most commonest type of parotid gland tumor ? a) Picin cell ??? b) Sorry I forgot Plzz red about it

60 y/o male known to have ( BPH) digital rectal examination shows soft prostate with multiple nodularity & no hard masses , the pt request for ( PSA) for screening for prostatic ca what will you do ? a) Sit with the pt to discuss the cons & rods in PSA test ???? b) Do trans-rectal US because it is better than PSA in detection c) Do multiple biopsies for different sites to detect prostatic ca

15. 16. 17.

pt want to quit smoking you till him that symptoms of nicotine withdrawal appear after a) 1-2 days ????? b) 2-4 days c) 5-7 days d) 8- 10 days

Plzzzzzzz

pt want to do dental procedure , he was dx to have mitral valve prolapse clinically by cardiologist, he had never done echo before what is appropriate action a. Do echo ???? b. No need for prophelaxis c. give ampicillin d. Give amoxicillin calvulinic Female postmenaupusal came with sign of osteoporosis,she afraid from vertebral stress fracture, best way to prevent: a) VD supplementation b) Diet c) excersise.

18.

Female com with lump in breast, which one of the following make you leave him without appointment ?

Cystic lesion with seruse fluid that not refill again?? Blood on aspiration Solid Fibrocystic change on histological examination

19. 20.

Infant newly giving cow milk in 9 months old , closed posterior fontanel, open anterior fontanel with recurrent wheezing and cough , sputum examination reveal hemoptesis , x-ray show lung infiltration , what is your action ? ): a) diet free milk b) corticosteroid c) antibiotics ?????? In battered women which is true: a) mostly they come from poor socioeconomic area b) usually they marry a second violent man c) mostly they come to the E/R c/o.. d) mostly they think that the husband respond like this because they still have strong feeling for them.

21.

Child has pallor , eats little meat , by investigation :microcytic hypochromic anemia . what will you do: a) Trial of iron therary b) Multivitamin with iron daily

.. malnutrition !!!!!

22. 23.

If we take Number 10 from all families to do a study ..this type is called ?? a) Randmized study b) Stratified Rand. Study a) Leave without TTT Child with hemagioma in the back ??

.42 .52

pt with recent Hx of URTI , develop sever conj. Injection , with redness, tearing , photophopia ? So, what is TTT

(= b) Intar-lesional steroid >> but Im not suuure c) Evacuation

a)Topical ABx )b Topical acyclovire >> mostly c) Oral acyclovire )d Topical steroid
)) ((

?? Which of the following increase absorption of iron a) Vit C >> correct b) Ca The End . . . .

.. .. . .. . .. .. .. . .. .. .. : Bader S. AL-Beshri ,Qassim COllege of Medicine . ..

1102-01-92 SLE Exam , Sat Done by : Sami Hillel AlHarbi ,Qassim college of Medicine

2-what is the defintion of standard deviation-measurement of variity 3- definition of positive predictive value-: -have the target positive risk

1-20yrs old age sexual active suffer from pain during intercourse and when do urine analysis was gram negative diplococci intracellular diagnosis is : -Genococcal sexual transmitted disease

4-Atrritable risk -: -measurement of exposed and not have the disease mius those exposed and have the disease

6-what is the side effect of fast correction of hypernatremia-? brain edema

5- 30 years female have abdominal pain non cyclic , no menorrhagia on examination have abdominal tenderness and painful cervix motion what is the diagnosis? pelvic inflammatory disease 7-contraidicated to use which of the following conditions is -? Iprofen -peptic ulcer disease

8-schizophrenia best indicator for prognosis-? family history of schizophrenia 9-female come with manfestations of hypothyrodism --sleeping,myxedema, cold intelorance , now she suffer from diffuclty he put a table show TSH=normal , T4 and... in breathing , wheezing ALP = normal normal, Ca =decrease , phosphorus = normal what is your diagnosis?

- secondary hypoparathyrodisim

10-old female came with scales aroud the areola ,she took steroid but no benefit on examination normal and no masses what is your next step? A-Antibiotics B-anti-fungal C- Mammography mammography ANTI-fungal

11- child come with hypetension , oligourea, cola urine .... diagnosis is nephritic syndrome what is the best next step : - urine analysisurine sedmentation microscopy 12-best treatment of bulimia nervosa cognitive behavior therapy 13-patient have ARDS now devolpe pneumothorax what is the cause central venous line 14-patient with Ischemic heart disease what it is best diet increase vegtable and fruit 15-newborn given an injection to stop bleeding Vit K Injection 16-best treatment of otits media senario Amoxcillin for sure Insha'Allah 17-patient come with diarrhea , confusion , muscle weakness he suffer from which ? A-hypokalemia B-hyperkalemia hypercalcemia A 18-cellulitis in neonate mostly caused by strptococcus B hemolytic 19-Patient come with jundice , three days after the color of jundice change to greenish what is the cause? oxidation of bilrubin

20-which of the following represent inferior MI I,II and AVF 21-Patient with Rhumatoid Arthritis he did an X-Ray for his fingers and show permanant lesion that may lead to premnant dysfunction , what is the underlying process? substance the secreted by synovial 100% correct -paget's best treatment for patient with HTN and DM is ACEIs 23-best treatment for female with migraine and HTN propnalol 24-which of the following drugs used in ischemic heart disease to decrease mortality ACEIs

25-defintion of case control study 26-60 year old male come with depressed mood , loss of interest , sleep disturbance after dying of his son 3 months back after long period of suffering of disease >>>what is your diagnosis Breavment

27-best way to prevent Entameba Histolical is boiling 28- prevention of lyme disease , what is best advice to parents insect invdelent 29-5year child obese best mangement family intervention 29- 12 year obese best way to reduce weight is decrease calorie intake 30-female suffer from post-partum physcosis best step after medication include family intervention on the mangement of disease

31-52 old female have ostopenia and she fear from getting back fracture best prevention is Vit D supplement daily 32-mid clavical fracture in neonate best descriptiom is usually heal without complications 33-fracure in the humerus affecting radial nerve lead to wrist drop 34-spondloysis and ulnar nerve comprssion mangement is ulnar nerve decompression 35-injury to the anterior of hand lead to can not opposition the thumb 36-best treatment of 4th degree hemorroids is hymeroidectomy 37-patient on wrfarin come with INR=7 what is your action stop warfarin and re check next day 38-which of the following associated with high risk of colone cancer high alchol , smoking , granunder disease 39-2 year old child come with brocholitis and cynosis best intial treatment is A-O2 B-antibiotics C-corticosteroids 39-best way to secure airway in responsive multi-injured patient is endotracheal intubation 40 child with moderate asthma and he on b2agnosit what you will add to decrease the recurence of asthma attacks corticosteroids inhaler 41-30-40 year male suffer from tetnus, vertigo , sensorconductal hear loss diagnosis is Miner's disease 41-patient 20 year old come with palptations ECG show narrow QRS complexes and pluse is 300 bpm what is the true Amidarone should included in the mangement Best wishes for all

40 years female complaining of thinking a lot in his children future, she is alert, anxious, cant sleep properly, poor appetite, she always make sure that doors in her home are closed, in spite of doors already closed, provotional Dx: MY SLE, 29/10/2011 OCD, GAD?????, schizo Long scenario of restless leg syndrome(he didnt mention Dx in scenario), 85 old male many times awake from his sleep bcz leg pain, this pain relieved by just if he move his foot, but it recure, atsetra,best management: Colazpin, haloperidol, lorazepam, one drug from dopamine agonist group forgot its name, its the right answer. best drug for von willbrand disease is:

fresh frozen plasma, cryoprecipitate????, steroids,(he didnt mention vasopressin in choices). Best fast management of acute hyperCalcemia is: Iv fluids, frusamide????, dialysis,

Scenario of digitalis toxcisity, old age male have CHF and he is on digitalis for 3 years, sudden decrease of visual acuity, he see yellow flash light. Bad prognostic Factor for schizo is : early onset???, family Hx, mood dis,

Null hypothesis definition>>> No ..No. Difenition of specificity: Without disease and _ve test.

Long scenario of a pt with melanoma in back, he is afraid of malignat change, which one of the following indicate malignant melanoma: >6mm, irregular and invade the skin, Regarding of DM in KSA: Most of NIDDM are obese, Best inv to visualize the cystic breast masses is: MRI, CT, Mammogram, US

Female pt new diagnosed as HTN pt, BMI 28, U sld advise her by: Body wight reduction ALONE doesnt benefit her HTN, wt reduction and exercise may benefit him,

Long scenario for pt smokes for 35 y with 2 packets daily, before 3 days develop cough with yellow sputum, since 3 hours became blood tinged sputum, X ray show opacification and filtration of rt hemithorax, DX: Bronchogenic CA, acute bronchitis???, lobar pneumonia Young male pt with dysuria fever and leukocutosis, PR indicate soft boggy tender prostate, Dx : Ac prostites, Ch. Prostites, prostatic CA

Pt with hypercholestrelemia, he sld avoid: Organ meat, Avocado, chicken, white egg

5 y child diagnosed as UTI, best inv to exclude UTI comp: Kidney US, CT, MCUG, IVU 6 y old boy, eat the paper and soil, best initial ttt is: Fluxitin, behavioral therapy,. Baby said baba mama, pincer grasp, creeping well, sitting wth support, estimate age: 6m,7m,8m,10m Overcrowded area, contaminated water, type of hepatitis will be epidemic: A,B,C,D One of these not live vaccine: HBV,OPV,MMR

Most lethal infection for a pregnant woman: Toxoplasmosis, HIV???, rubella, measles Drug of choice for gonorrhea: Ceftriaxone, ampicillin, cipro Drug of choice for a schistosomaisis is: Praziquanetil, oxaminiquine, artemether (sure)

Wavy like abdomen: Bowle obs.

Pt with acute Periumblical pain with +ve obturator sign, -ve psoas sign, Dx: Ac appendicitis, Ac cholycystites, Ac pancreatites

Old male pt, smoker, alchoholic, fatigue, debilitated, back abd pain (scenario didnt mention to jaundice or lab findings) Dx: Ac pancreatitis, ch pancreatitis, pancreatic CArcinoma???, insillinoma ( Bcz he didnt mention to jaundice or decurvesier low, Im against CA ).

55 ys old male pt, presented with just mild hoarsness, on exam, there was a mid cervical mass, best inv is: Indirect laryngioscopy, CT brain, CT neck Varecilla vaccine: 2 dose 6 weeks apart Pt with +ve tinnle`s test, best position for a hand fixation: planterflexion, dorsiflexion, extension. ( suuuuuure )

45 years male, presented with back pain radiate to the lower, on exam just a paraspinal muscles were contracted and tender, on MRI, diagnosed as a cervical stenosis, one of the treatment method is(didnt say the best) : physiotherapy, cervical laminotomy,. (suuuure). Female postmenaupusal came with sign of osteoporosis,she afraid from vertebral stress fracture, best way to prevent: VD supplementation, diet, excersise. ( didnt mention to HRT, if mention; it will be the answer ).

Verrryy long scenario of old age pt with DM, HTN, hx of multiple cardiac attack, CVA, came for routine check up in PHC, u found bilateral opacification in both lenses, with decreasing of visual acuity, u will: Refer to lazer therapist, refer to cataract surgeon, refer to ophthalmologist, follow up Best preventive method for lyme disease: Insect repellent, wear fiber clothes, (suuuure)

45 male pt, will undergo dental procedure, he had a Hx of LBBB, but there is no structural abnormality, and ejection friction rate in normal range, no hx of arrhythmia, the good next step: Avoid hard dental maneuvers, give Abs before, give Ab after, no need for prophylaxis???. ) ( 4 years old child, was diagnosed as SCD,so many times came to hospitals with, dyspnia, dactylites , ( he put sign of acute crises ), the best strategy for prolonged therapy is: IV hydration fluids with analgesia???, follow in Out pt clinic, refer to tertiary haem center. ( bcz he said STRSTEGY, im going with last one). 80 ys old male pt, come with some behavioral abnormalities, annoying,(he mention sm dysinhibitory effect symptoms) , most postulated lobe to be involved: frontal, parietal, occipital, temporal. Long scenario for a pt came to ER after RTA, splenic rupture was clear, accurate sintences describe long term management:

We give pneumococcal vaccine for high risky people just, we sld give ABs prophylaxis if there Hx of contact even with vaccination against pneumococcal, pneumoccal vaccinw sld not be given at same time with MMR 40 ys female, multigravida, no sexual intercourse for 1 year bcz her husband going abroad, C/O was intermenistrual bleeding with menorrhgia, provotional Dx: Endometriosis, Endometrial CA, ch endometrites??? Female pregnant, 32weeks of gestational age, diabetic, and she has a Hx of full term fetal demise, but her DM now well controlled, and BPP show no fetal distress,best management: Wait for SVD, report a CS in 36 weeks?????, Teminate pregnancy Couples with 6 month of unprotictive sex, no pregnancy: Wait , do semen analysis, US

Pt with a long Hx of conjuntavites, red eyes, corneal ulceration, most common cause: Ultraviolet rays, dust and pollens, viruses (suure) Typical scenario of osteoarthritis. Dr. Mohammad Sa`d Alhomoud Ur praaaays Best wishes for alllll <3 Triad of heart block, uveites and sacroileatis,Dx: Ankylosing spondylites, lumbar stenosis, multiple myeloma

SLE- exam - Abdulaziz ALGharis,Qassim College of Medicine 1. Patient with hypertension , DM, somking, which the following are most important to be deal with : a. obesity and HTN b. smoking and obesity c. smoking and HTN 22-11-2011 2. Best treatment for depression>>> ssri 3. Mechanism of action of SSRI is ? i a. Increase availability b. Block receptor c. Decrease availability

4. Contraindication to use in Migraine : Buprobion, lithium, valium ,

5. Patient with come decrease hearing, ear examination lost tympanic membrane normal looking , which one is most likely cause : serous OM, acute OM

6. Patient has decrease visual acuity bilateral, but more in rt side , visual field is not affected , in funds there is irregular pigmentations and early cataract formation . what you will do a. Refer to ophthalmologist for laser therapy

7. Female come to with 3 UTIs history in last 6 months,, what is your advise? a. wipe frome behinde to front after defecation b. take a bath insted of shaower. c. increse flude intake

b. Refer to ophthalmologist for cataract surgery ( true)

8. Heavy smoker came to you asking about other cancer, not Lung cancer, that smoking increase its risk: a. Colon b. Bladder c. Liver 9. Carbl tunnel syndrome , in which postion would u fix patient's hand>>> dorsiflexion ( sure) 10. Pt . 32y- have 2 children ,done a pap smear that showed atypical squmous , what it is the next step: a. Cone biopsy ( I think it's wrong ) / Direct biopsy / 11. Pt. has columnar change in esophagus + finger projection >> adenocarcinoma, barrot esophegous, SCC 12. Breast examination to young girl : Mammography and self comp. 13. Positive prognostic factor in schizophrenia >> prominent of affect symptoms 14. Pain near eye precided by tingling and parathesia ocure mant times a week in the same time , also there is nasal congestion and eye lid edema dx? a. (Cluster headache)/migraine with aura/ tension headache / withdrawal headache 15. SCA with hip joint pain 4 weeks >> AVN 16. True about peritonitis>>> chemical erosion 17. A case of how to mange the enuresis : ?? I don't know

18. from surgery the discharge is greenish showed Gram +ve cocci the organism is: a. Clostridium b. Streptococcus c. Pseudomonas 19. 43 yr old sustained trauma to the chest present with severe short of breath with cyanosis, his rt lung is silent with hyperresonance. The FIRST step to treat this pt: a. O2 mask b. Tube thoracostomy c. CXR 20. computer programmer presented with wrist pain and +ve tinnel test. The splint should be applies in: a. dorsiflexion position b. palmarflexion position c. extension position 21. Infant born with hemangiomas obstructing his vision to prevent Amblyopia the child need to undergo the surgery within: a. After 1 day b. After 1 week c. After 1 month 22. 17 mitral stenosis : a. A diastolic high pitch b. B - systolic low pitch c. C- diastolic low pitch 23. human bite to the hand .. greatest risk of infection in which posetion? a. dpendent b. clenched 24. ??.. a- PCOS b- Asherman syndrome

infertile pt. with 3 previous d/c .. otherwise healthy .. Dx

25. female infertile, hirsutism, male hair, plvic exam normal ,no US mentiod ddx: a. hyperprolactin, PCOS,

26. -13healthy child with pRBC in urin 15 cells/hpf .. what to do ??.. tricky quastion a. repeat urine analysis for blood and proten. b. cytology ( I think wrong ) c. renal biopsy d. serm parathyroid hormone 27. 28. a. b. c. d. e. SE of steroid : DVT , osteomalacia Lactational mastitis..Rx: _ doxycyclin _ ciprofloxacin _ ceftriaxon _ gentamyecin _ cephalexin

29.

30. Snellen chart : 6 meter 31. What to adjust ? Insulin chart : long night, short day 32. Athelis + groin rash >> topical antifungal, Abx ? 33. Which drug contra indication in cluster headache ? buperbion, lithum, valium, 34. Pt daily asthma , nothing at night, using herbal for 2 months with no improvement : inhalation salbutamol, high dose steroid inhaler, ipratrubium 35. How to manage golf elbow : golf refine (x) , I don't remember answer

Gout : sodium urate or Calcium pyrophosphate

36. Children has bitten patient in back, child immunization up to date, Rx: suture, tetanus toxoid ? 37. Stroke >>> HTN 38. Length of spinal cord post puberty ? 1,2,3 yr 39. Burn @ face and neck ? admit hospital ?? 40. L4/5 disc prolapse will cause : knee parasthesia, ankle jerk, groin sensation 41. Vitamine cause loss sensation and momvet disorder: Vit B12, B1,B2 , niacin 42. MMR and lactation is :live attenuated virus 43. Pt complain of Dysmenorrhea + Amenorrhea >> Endometriosis, Endometritis, Polyp 44. Migrane, ? S/S unilateral pain, photophobia 45. Post menopause Female with lower back pain , doctor said it's early osterprosis , she is not on estrogen medication , what to give ? VIt D + Ca+Bisphonate , Floic acid and ., others 46. Old man with unrine incontunance, he feel distended bladder : stress, urge, flow , 47. Elbow truma >> posterior pad sign 48. Firgen body fully removed from eye, what to give ? Topical Abx, topical steroid 49. Risk assessment of ve true !!! ?? I choose not predict of predicted ! 50. Lichnus planus ( photo) flat top papule, purple. 51. Boy eaten family medication come with black vomit and other sign>> Iron intoxication 52. Case of spontnus pnumothorax 53. What is sign of high aldosterone? Hypo-K 54. Pt has high cholesterol, Jaundince, xanthoma, xantholesema , DDx ? primary bilary cirosis 55. 10 Kg boy, maintaince is 1000 mL 56. Young, drug abuser, asymptomatic . What to investigate ? HIV, HBV, St.viredans 57. Varicolla vaccine , 2 dose, 6 weeks apart 58. Pt complaining of pain at neck, Xray show narrow space of verbtebra and ostephyte : Ankylosisng spndiolytis 59. Boy wet bed , how to treat ?

60. Sexually active multigravida, joint pain, swollen and red >> septic arthritis ? 61. Woman with tumor at pelvic examination, what next ? cone biopsy, direct biobsy, . 62. What vitamin given to neonate to prevent hemorrhage : Vit K 63. Hypercholesteremia should avoid : Avocado

SLE- exam - Muzoen ALMohamid ,Qassim College of Medicine 26-11-2011 -1typical presentation of diabetic nephropathy , when

- c-20 b-15 a-10

copd patient not responding to bronchodilators .. what u will add to 2-: him a- aminophylline I.V ( ?? cord prolapse Varicella HSV b- methylprednisone I.V 3- : instrumental delivery , what u should take care about

child with feve , anorexia develop papuls then became vesicles then -4pustules in hourse

a-screening for breast cancer decrease

-5: what is most true about eduacation

b- screening for colorectal cancer still low d-screening of lung cancer decrease

physician screening fo smooking is satisfactory 6-patien complain of painfull knee ,, (before 4 months the RT big toe was swollen and panifull )the aspiration from knee was negative for crystals a- ca pyrophsphtase crystals after acute episode b- uric acid after acute episode d ca pyro. After perci. Superaturation st meconazole cream estrogen cream douch c- uric acis after perciptation of supersaturation state 7-female c/o colorless itching vagina , her partner c/p urethral disch. : . Cervical examination shows strawbery spots progesteron cream 8-? ttt of tremor and sweatinf in front of audiance Propranolol ssri

9-four y old child c/o bleeding for 4 m, intermittent painless bowel movement , good appeite , after ex. The examiner found blood at his : digits

UC

meckles diverticulus juvenile polype Malaria 10-? fever , rigor for 3 days .painfull hepatosplenomegally vesiral leishmania toxoplasmosis

11-: stable non bleending hemhrroid increase fibre intake diabetic pstient with ulcer in foot , not healing , not infected , high ? blood glucose a- high blood glucose stimulate bacteria to grow b- decrease phagocytosis c- dec. Immune system

12-: appropriate way to prevent spread of diseases change human behavior screening tests

13-: to prevent trachoma

!! water and eradication Milk caries

water and sanitation

m boy with dental caries at icisors and central tooth .. what is the 18 14- cause excess fluride m12 m18 m36

15- first few words at

SLE- EXAM FOR hussam alsulmi and abdulrahman almoklef,Qassim college of Medicine Written by : hussam : What is the Pathology of coronary heart disease

Atherosclerosis in the coronary arteries Atherosclerosis in the arteries

Deposition of fat in the outer layer of aretery

Batterred ( same choices in qassim collection

In battered women which is true: a. mostly they come from poor socioeconomic area b. usually they marry a second violent man c. mostly they come to the E/R c/o.. d. mostly they think that the husband respond like this because they still have . strong feeling for them

Female patient presented with tender red swelling in the axilla with history of repeated black head and large pore skin in same area: ttt is a. Immidate surgery b. Topical antibiotic c. Cold compressor d. Oral antibiotic Which of the following oral contraceptive drugs cause hyperkalemia: .. "" Norethadion ..etc Ehinyle seradiole

Patient with hypertension , DM, somking, which the following are most important to be deal with : obesity and HTN smoking and obesity smoking and HTN Children while he was playing a football , the ball hit his hand from lateral fingers, after a while the children complaing pain and swelling on those fingers and painful middle finger with hyperextension of interphalengial joint, swelling was more in the DIP and IP Joints , also , there was pain on his palm, what is the most likely cause: Rupture of profound ms in hand( 100% true) Rupture of superfacial ms

Intra articular interphlengial joint fracture Extra articular.. Patient was presented by bollus in his foot , biopsy showed sub dermal lysis , fluorescent stain showed IgG , what is the most likely diagnosis : A. Bolus epidermolysis . B. Pemphigoid vulgaris . C. Herpetic multiform . D. Bullous pemphigoid . Female patients come with Black maculae in forehead and cheeks. She is Not pregnant what is the most likely DX: ( I forgot if it was associated with sun exposure or not) Lentigo PeutzJeghers syndrome, Chloasma. Dermatitis herpitiforms Case of old man presented with Sing s of depression after died of someone for 6 wk ( long scenario made me hesitated in pick up the correct answer) ( a very short scenario)Female come to the clinic with her baby of 6 month , she had tremor and other sign I forgot it, which of the following is most likely dx The correct answer is d

---bereavement , adjustment with depression . Depression

hypertyrodism hypothriodism

sub acute tyroditis

Postpartum thyroiditis ...Hashimoto

Why do we take a prophylaxis in asthma, is IT to prevent:

To treat Allergic sensitivity To Decrease inflammation etc Cefurxim , cipro, Doxo Old pt with pneumonia , H. influenza .. Which of the following is the best in Ttt:

. Penicillin

ceftriaxone

VSD

Left ventricle hypo plastic ,

Infant come after 5 wk with difficult breathing and occuasionly turn to blue. Pan systolic, most likely DX: Gardener complaing of Watery discharge from his eyes , and he can not avoid his job , how would you solve this problem :

Cool compress ( sure) saline venger lenses.

Female had history of severe depression, many episodes, she got her remission for three months with Paroxitine ( SSRIs) .. now she is pregnant .. your advise a. Stop SSRi's because it cause fetal malformation b. Stop SSRi's because it cause premature labor c. Continue and monitor her depression d. Stop SSRIs The correct answer is c or a

Postpartume blues ,pp pychosis pp depression

( very short scenario) Femal after delivery come with tearfull and irritability , the mood is normal

Best drug to treat depression in children and adolescent is: a) Fluxetine (Prozac)

In general, SSRI have the least side effects then other classes of antidepressants. Some SE: Sleep disturbance, dizziness, sexual dysfunction, Nervousness, and diarrhea

RT -ventricular hypertrophic and dec pul pressure Lt atruim hyper and dilatation( ithink true) Rt ventricl hypertrophic with constrict champer Lt atrium hypertrophy with constrict champer Wich of the Following true about child spell

Buprobion >> side effects ( same question in Q- collection) Patient with rheumatic hear disease and he developed mitral stenosis , what most likely will happen to the heart:

Picture of graphical presentation chart" simple columen" in which the X= Osteioporsis AND Y= Age,, chose the best statement that explain the chart"

Patient with Sever hypothyriodism and hyponatremia (108= Na ), high TSH and not respond to painfull stimuli, how would you treat him :

Oral intubation , Thyroid replacemet , Steroid and 3% Na Same above but Without steroid Thyroid and fluid and%3 Na Bcg ( Pneumocool Thyroid and fluid replacements only

In close contact with asymptomatic TB what you will give 94. Female com with lump in breast which one of the following make you leave him without appointment ( ) Cystic lesion with seruse fluid that not refill again?? Blood on aspiration Solid Fibrocystic change on histological examination ??? 95. Patient with come decrease hearing , which one is most likey cause : 96. Pat with perennial allergic rhinitis. Tttt

Acut Oriya media

Steriod ( I think true) anti hisatmin Decongestant

serous om.( I think true )

Patient has decrease visual acuity bilateral , but more in rt side , visual field is not affected , in fundus there is irregular

Allupurinol ( I think true) intakeclaries Propenside Epi dural abscess Spinal abscess

Increse fluid

Patient is a known case of gout presented with frequent Stones .. Increased creat and urea .. The time btw attacks is decreased , how would you decrease the frequency of attacks

pigmentations and early cataract formation . what you will do a) Refer to ophthalmologist for laser therapy b) Refer to ophthalmologist for cataract surgery ( true)

Old patient presented with Ear pain ,headache , hem paresis>> most likely cause:

Subdural hematoma

Which of the following is true regarding gastric lavage : A. Patient should be in the right lateral position . B. It is not effective after 8 hours of aspirin ingestion . a. Mostly occurs between age of 5 and 10 b. Increase Risk of epilepsy

In holding breath holding which of the following True: c.Aknownprecipitant cause of generalized convulsion d. Diazepam may decrease the attack

patient presented with excruciating headache and blurred vision: Mother worry about radiation from microwave if exposed to her child. What you tell her: a. Not all radiation are dangerous and microwave one of them b. Microwave is dangerous on children c. Microwave is dangerous on adult d. --- Woman with cyclic bilateral nodularity in her breast since 6 month, on examination there is 3 cm tender mobile mass in her breast : what you will do next e. FNA with cytology f. Mammogram g. Biopsy h. Follow up for next cycle i. Observation Male patient have ARDS &on ventilation have manifestation of pnemothorax<<<-ve pressure ventilation/lung damage / central line insertion Heavy smoker came to you asking about other cancer, not Lung cancer, that smoking increase its risk: a. Colon b. Bladder c. Liver How to prevent lyme disease: Insect repillant/ change the cloth to the natural clothes Wound at end inflammatory phase when: a. Epithelial tissue formation b. Angiogenisis c. when the wound clean d. Scar formation acute angle glaucoma

8y with weight and height > 95%and BMI= 30 .. What u will do for him:

Observation for 12 month /refer for surgical intervention/ life style modification/ What it is the most common congenital heart disease come with rheumatic heart disease: VSD/ASD/coartication of aorta Child with failure to thrive , pink, O/E thereis pansystolic on the lower sternum ..DX VSD/ TOF/TGA/ pulmonary stenosis with left ventric../ tricusp atresia Patient 57 y-o, somker for 28 y , presented with bleedind per rectum and positive guiaic test , also he has IDA:

COLON CA/ IDA

Antihistamine( 100 % sure) Uvietis ( sure 100%)

Pt, presented with sever ingestion of eye and watery discharge ,, no history of itching , truma and pain ,, vision is normal , what most appropriate ttt: Pat presented with constricted pupile, ciliary flushing and cloudy anti. Chamber there is no abnormality in eye lid, vision and lacrimal duct :

Pt he did not see the traffic light clearly , what u will do for him >> snelling chart test ( sure Pat know case of asthma , and allergic rhinitis , he presented with with excuriations , dry skin and pruritis

Chlesteatoma/ BPPV/Vestibular neuritis( true)

PATIENT he can not opposite his thumb >>> median nerve ( sure) Carbl tunnel syndrome , in which postion would u fix patient's hand>>> dorsiflexion ( sure) Pat presented with sharp sever chest pain increase with movement and supine postion and decrase in leaning forward ( also there was a pic of CXR)>>> Pericardial effusion A clear case of primary dysmenorrheal ( 19 y-o female presented with pain) Patent with amenrrhea for 2 month , on exam there is tender pelvic, prolactine was normal ,, what it is the most appropriate invest .? >>>> US ?? Pt . 32y- have 2 chidlren ,done a pap smear that showed atypical squmous , what it is the next step: Hysterectomy / repeat after 1y/ loop elec / coploscopy Pt . complaine that he has a palpitation , sweating and panicky feeling when there are talking in there job ,,, but there are doing well in the job,, dx? Agoraphobia/ GAD/performance anxity ( true) Best ttt for depression>>> ssri Indication of ECT >>> Major depression with psychomotr symptoms( sure) Positive prognostic factor in schizophrenia >> promenant of affect symptoms Loss of smell >>> temprol lesion Pt . had a closed head injury after that he can not eat by using spoon? Lesion Cerebellum ?? Patient had URTI then he developed vertigo what it dx?? Pat has snoring in sleeping and and on exam there is larg tonsile >> what u will do for him :

most likely DX >>> ATOPIC DERMATITI ( SURE)

Weight reduction/ adenoidectomy

Cluster headache/migraine with aura/ tension headache / withdrawal headache US/ MRI/ mammogram

Pain near eye precided by tingling and parathesia ocure mant times a week in the same time , also there is nasal congestion and eye lid edema dx?

What is the most appropriate investigation to visualize the cystic mass :

Hospitalization and pain manegment and observation Give him narcoyics

Outpatient manegment hydration , pain mangment and observation

Pt with hx of SCA.. he admitted many times to hospital due to crisis attacks : and know he caome with abdomenial pain and neck , body and arm pain ,, what u will do for him:

True about peritonitis>>> chemical erosion What is true about appendicitis in elderly patients>>> rupture is common Which of the following is the most important prognostic factors in CML: Stage/ age/ lymphocytic dubling time/ involvement of bone marrow degree Pt has pharyngitis rthen he developed high grade fever then cough then blateral pulmonary infiltration in CXR ,,, WBC was normal and no shift tp left: dx( organism) Satphylococcous aureous/ staphylococcous pneumonia/ legonilla / clhlamydia PATIENT has RTA and membranous uretheral disruption . Long scenario: Suprpubic catrheter ( may be ) / pubic repaire/ trans uretheral catheter/ abdominal repaire

Will you give him prophylaxis before procedure/ u will not give him 2g amoxicilline before( true) / 2g after Patient with hx of endocarditis and he will do an oral surgery :

Patient with testicular mass . non tender and growing on daily basis . O/E epidydmeis was normal.. what u will do: Refer pt to do open biobsy or percutaneous biobsy / refer him to do US and surgical opening ( I think true) Newborne with white creamy leasion on the mouth after taking course of antibiotic ,, ttt: Oral nystatine ( t)/ steroid/ AB/ antiviral A case of how to mange the enuresis : Imipramine and vasssopressine( I think true)/ clonodine and vassopressine/ clonodine and guanfacine PPV ; Proportion of people at high risk which have target population To prevent infection in neonate >> wash hand before and in between patients examenations Patient is known case of cervical spondylolysis , presented by parasthesis of the little finger , with atrophy of the hypothenar muscles , EMG showed cubital tunnel compression of the ulnar nerve , what is your action now : C. Ulnar nerve decompression . (100 %sure) D. Steroid injection . E. CT scan of the spine . Patient with subaortic hypertrophy:

CBC/ urine preg test/ US/MRI/blood groubing and Rh

Patient with peptic ulcer and he seek for medical dx: Endoscopy/ . /.. Early pregnant come to your clinic, which of the following is most benefial to do :

Continue and check after 1-2 month/ decrese the dose/ stope until tsh is become normal Incrase dos and after 3 wk/ incresea and follow after 6 wk Same the above case but : Most potentional prevention to decrase the disease>>> immunization 70 y-o pt , come with investigations showed osteolytic lesion in skull, monoclonal spike, roleahex formation>>>> multipul myeloma

A pregnant lady came to you to in second trimester asking to do screening to detect down syndrome, what is the best method: Triple screening( I think true)/ amniocentsis/ Ttt of trichomoniasis >>> metronidazole Patient with gunshot and he developed dypnea , rasied JVP , Deviated trachea ,>>> ttt >>> needle decompression 140/80 and RR= 18 ,and there is part of omentum protruded,, what u will do : Exploration of the wound/ schedule for laparotomy( I thing is true) / DPL" diagnostic peritoneal lavage/ FAST Picture of slightly red swelling just above the nail bed of finger , painfull, patient is using augmentine for 1 week with no benefit, what you wiil do: Incision and drainage with general anestheasia/ I and D with local anestheasia/ change AB/ Complete augmentine for 1 wk Pt known case of hypothyrodisim , and you start levothyroxine but she come after 1 wk with cold intolerance, and bradycardia, THS INREASED ;

by SLE : 27/11/2011 Done by : thamir als3eed 1- normal puriparium ()

mother start breast feeding after 24 hours lochia 2-indication of adenoidectomy sleep apnea abscess

epidural anesthesia normally cause urinary retention

ulnar nerve injury ---> wrist drop 3 radial nerve injury --> claw hand

median nerve injury --> atrophy of interossus muscle tarsal n injury --> tarsal tunnel syndrome(correct)

positive menngiocoal tb 6- girdasis-5 - rifampicin 7 days - 7- about ocb :

: delsion-4

3-single dose IM ceftriaxone a- decrease breast cancer

b-decrease ovarian cancer (my answer) c-increase ectopic pregnancy d-don't take by diabetic patient e-dnt take by healthy women over 30

8- ulcerative colitis in compare to chron disease a-fistula b-risk of cancer 9- old age , smoker ( ) obese , intermittent diarrhea , bleeding per rectum , positive Stool guaiac test , IDA ) (

10-about ecg

p waves are caused by atrial depolarization

) ( colorectal ca

- ventricular dysfunction

,drolling , , , - 5 years , : :, pharynx is congested and filled with mucus croup acute epiglottis

, compressible , no , , 13- adult , ,erythema -1 aspiration biopsy - synovial analysis wbc 15000 or 150000 o_O synovial analysis

http://en.wikipedia.org/wiki/Synovial_fluid

-51 Proctalgia fugax , , base skull fracture -61 ?anterior 2/3 , which nerve affected facial nerve -71 , , , no change in conscious , ct mri observation refer to neurologist -81 1-oral ab 2-local ab 3-oral steroid 4-local steroid ab ) -91 ( discharge , conjctivits :[ ( , watery) , discomfort sensation ? , visual acuity 1-bactrial conj 2-virl conj 3-chlymdia conj 4-glucoma kawsaky -02 strawberry tongue : 21- inferior infarction changes in leads II, III, aVF tempro-frontal -22 frontal )androgenic alopecia (sure

23- scale ! !! - outbreak for tb , , rifampicin - bcg vaccine

- - DKA 1-starvation cause increase of amino acids and fatty acids which utilize by the body Ketone body which excreted in urine 2-decrease in insulin lead to > fattyacid > ketone body 26- tight band like dka 1-migrane , stress , Bursting throbbing

2-tension headache

-27 trauma in chest present sob with cyanosis, his rt lung is silent with hyperresonance. The first step to treat this pt: a. O2 mask b. Tube thoracostomy

c. CXR

d-needle decompression : specfity - :129- DM 4DR

- single palmar crease ) 12 (trisomy

SLE- EXAM FOR Moyad ALKhalifa ,Qassim college of Medicine # which of the following true about headache :" -increase ICP at last of day

-normal CT may exclude subarachnoid hemorrhage . ???? -amnursus fugax never come with temporal arteritis . - neurological sign may exclude migran -cone biopsy -pap smear

#patient came with cervical carcinoma next investigation : - Direct biopsy

# patient with typical sign of infections mononucleosis come with abdominal pain and hypotension next step : -abdominal CT and IV fluid ???? -antibiotic and IV fluid and and observation Herpes zoster

#patient with hypersensitivity skin at back take paracetamol and develop vesicle at back extend to abdomen Dx :

#infant (28 week gestation ) 900 gram go to NICU after resuscitation ABG increase of Pa co2 with normal PH next step :

-give IV acyclovir -give IV bicarb - give IV glucose

#picture of viral warts -pleural calcification

#pt with asbestoses what is the specific sign : #dust mite how to prevent : -keep the house humid -wash clothes with hot water #pt with bilateral infiltration in lower lobe (pneumonia )which organism is suspected : -ligonella ( my answer ) -klibsella

# pt in crowded area and has pneumonia which vaccine you will give (long scenario) -hemophilus influenza (my answer ) -menigococcal vaccine -smoke inhalation . #pt in burn wll die due to :

#what vitamin you will give to prevent hemorrhagic disease of newborn : Vit k #pregnant with thyroid function test and it is completely normal except high TSH ..what diagnosis : -Due to pregnant(my answer ) -compensated euthyrofism . #holding breath :

-risk for generalize convulsion -IV valum

#elderly patient bedridden for long time what will you do : -include family support (my amswer ) -direct question -fluxitn (SSRI) #you have difficulty to get information from patient ..next step: #young patient with anxity ,,TTT : #which drug contraindication in peptic ulcer :

-drug related to (NSAID) but I forget exactly ^_^ #degree of scoliosis to refer to orthopedic :

-20 degree

#pt with dysphagia , weakness ,fasciculation ..: -motor neuron disease ??? -polyneuropathy

#typical scenario of osteoarthritis

#typical scenario of rheumatoid arthritis -prednisolone -acyclovir -antibiotic

#pt with polymyalgia rheumatic treatment : ( I advise you to read about it )

#typical scenario of bacteria vaginosis : -fish odor discharge , clue cells . #treatment of thyroid carcinoma : -surgical resection (my answer ) . -radiotherapy -antithyroid drug

#pt discharge with menigiococcal meningitis and now asymptomatic ..what is next step: -rifampin (my answer ) not sure until now ..correct or not -ceftriaxon -no vaccine -CT scan

#high sensitive and specific test for renal stone : #pt with nodule near to eyes (dacryocystitis ),,TTT: -oral antibiotic

SLE- EXAM FOR Mohammed Al-Anazi,Qassim college of Medicine 20-11-2011 TTT of refractory hiccup? *Best TTT of somatization? Multiple appointment ,multiple telephone calling, antideppresant,send him to chronic pain clinic. *carpel tunnel syndrome >> hand position *why influenza vaccine given annually? Bacterial resistance ,viral antigenic drift . *visual acuity >> snellen chart *cover one eye another eye go laterally >> strabismus , amyblobia,3rd nerve palsy *TTT of miagrine? Sumatriptan *most common cause of intracerebral Hg >> HTN *most common psychiatric condition come with mania ? paranoid,grandiosity *most common cause of female precocious puberty? Idiopathic *twins male and female 13 years old what you will tell the family? Female puberty 6-12 months earlier to male ,2-3 years before male,same age of puberty ,male earlier than female. *patient come with difficulty breathing in one nostril ..O/E:erethematous structure best TTT: decongestant, antihistamine, sympathomimetic *patient take seldenafil which drug must be avoided? Nitrate *estrogen containing pills associated with ? breast ca, endometrial ca, ovarian ca, thrombophelebitis. *pregnant want to take antibiotic ,not known to sensitive to any drug , which antibiotic safe to given to pregnant? *patient with fever and fatigue prior to develop maculopapular then vesicle and pustule ?HSV1,HSV2, varicilla *contraindication of breastfeeding ?HIV, hep C *best stimulus for lactation? Breastfeeding!!

*Diagram and answer was >>patient above 80 years at high risk of osteoporosis. *tibial tubercle pain?osgood shlutter, perthes disease. *typical scenario about osteoporosis. *which is the following true about chronic fatigue syndrome? Give him antidepressant, rarely resolve with TTT. *which is of the following true about pathophysiology of HTN? Decrease sensitivity of baroreceptor, peripheral vascular resistance , fibrinoid change of the vessles. *10 year old had an episode of rheumatic fever without any defect to the heart. The patient need to take the antibiotic prophylaxis for how long: a. 5 months b. 6 years c. 15 years *pt discharge with menigiococcal meningitis and now asymptomatic ..what is next step: -rifampin -ceftriaxon -no vaccine *postmenopausal women at high risk of: osteoporosis *why SSRI best TTT? Effective and tolerable *compliance of prophylactive antiasthmatic drugs important to reduce :airway inflammation, reduce esinophil

*patient blood group A, they gave him blood group B and developed limper pain, dyspnea and hypotention why? Q was about mechanism *quick TTT for SVT? Adenosine!! *case of appendicitis do resection what you will find if you examine appendix histologically?normal, lymphoid in muscular layer, neutrophil in muscular layer *Pt. Taking bupropion to quit smoking, whats the side effect? A. Arrhythmia B. Seizure

C. Xerostomia D. Headache *Mother has baby with cleft palate and asks you what is the chance of having a second baby with cleft palate or cleft lip , 25% 50% %1 4% *Which of the following dietary factors considered protective against cancer Vit D Fiber Calcium *Q about Buchard's nodule in PIP *2 months amnorrhea refuse examination because she is tense and anxious what will do for her : FSH and LH, US pelvis *stab wound of abdomen and omentum is seen what will do : exploration, scheduled lapototomy *vit C in wound healing? Collagen synthesis *60 years old patient has only HTN best drug to start with: ACEI, ARB, diuretics, beta blocker, alpha blocker *most common cause of nephropathy : diabetic nephropathy *baby with clavicle FX? Figure 8 !! *self examination of breast every? Weekly, monthly, 6months, yearly *drug induce lupus ? hydralazine *typical case of Wegener with saddle nose and other feature. *pregnant I forget GA but in second trimester with uterine fibroid come with abd. Pain what will do for her : myomectomy, drugs, terminate pregnancy *Battered women. *structure normally not palpable? LN *case AF best TTT? Digoxin, synchronized DC *child know 4 color and can wear clothes with little difficulty what is the age? 48months, 36months *Q about female urine incontinence ?

SLE 26 nov 2011 Abdulrahman Al-Darrab King Saud University 1- How to prevent malaria: a- Kill the vector and avoid mosquito bites b- Kill the vector and spray your clothes c- Avoid and spray d- Something I searched it ..and most likely the answer is A 2- Asbestosis : a- Bilateral fibrosis b- Plueral calcefication The most likely answer is A 3- Surgery in c3 colon cancer : a- Curative b- Palliative c- Diagnostic The answer is A

4- Weiiiiird psychiatric scenario .. patient with echolalia, echopraxia, poor hyegine, insomnia, and weird postures what's the treatment? a- Lithium

5- (picture) showing huge mass in the Rt side of the neck with normal skin color .. no other masses in the body and some signs : a- Tb b- Infecious mononeoclusis c- Lymphoma The most likely answer is C 6- Long scenario for a lady suffer from vulvar itching .. remember that there's "bubles" in the scenario .. what's the dx: a- Lichen simplex chronicus This is the correct answer 7- There is interaction between Carvedilol and : a- Warfarin b- Digoxin c- Thiazide d- Something 8-

9- Scenario .. pt. suffering from wheezing and cough after exercise .. not on medications .. what's the prphylactic medication ? a- Inhaled b2 agonist b- Inhaled anticholinergic c- Oral theophilline 10Which of the following doesn't cause ear pain ? a- Pharingitis b- Otitis

groin and some other symptoms .. what's the dx: a- Mesentric ischemia b- Renal colic The most likely answer is B

11Scenario .. 18 months has dental decay in the upper central and lateral incisors .. what's the cause of this caries ? a- Tetracycline exposure b- The family doesn't brush his teeth (something like this) c- Milk bottle

c- Dental caries d- Vestibular neuritis The correct is D

12Baby with apgar score 3 in 1 minute ( cyanosis .. weak breathing .. pulse of 60 ) .. what's the best thing to do? a- Warming and drying b- Ventilation

13Old patient .. stopped smoking since 10 years suffering from shortness of breath after exercise but no cough and there was a table Fev1=71% Fvc=61% FEV1/fvc=95% Tlc=58% What's the dx? a- Restrictive lung disease b- Asthma c- Bronchitis d- Emphysema e- Obstructive with restrictive

14Scenario .. diffuse abdominal pain, diminished bowel sounds .. x-ray showed dilated loop specialy the transverse .. what's the dx? a- Acute pancreatitis b- Acute cholecystitis c- Bacterial enteritis 15Scenaria .. child sweats at night .. myalgia . arthralgia .. pericarditis .. what's the dx? a- Kawasaki b- Still's disease Most likely A 16Scenario .. patient with multiple pigmented spots and other symptoms .. +ve family Hx .. what's the dx? a- Neurofibromatosis b- Hemochromatosis Most likely A 17Question about spontenuos abortion : a- 30-40% of pregnancies end with misscariage b- Most of them happen in the second trimaster c- Cervical assessment must be done

18Propythiouarcil mechanism of action : a- Inhibits release of thyroid hormone fortha gland b- Inhibits releaseor hormone for thyroid globuline c- Something about inhibt coupling and something about tyrosine

19Cause hypertensive crisis: a- Enalapril b- Lorsartan c- Hydralazine The correct is C

I thimk the correct is C

20Question about ptrygium a- Can cause vision impairment This is the correct

21Long scenario about h pylori .. what's the regimen? a- Omeprazole, clarithromycin, ampicillin This is the correct

22Colzapine is used in which childhood psychiatric disease? a- Schezophrenia b- Depression c- Enuresis 23Hypertensive patient with liver cirrhosis , lower limb edema and ascites .. what to use ? a- Thiazide b- Hydralazine c- Something 24Scenario about Acute pancreatitis .. how to feed ? a- TPN Correct

25What's the most common area in women gonorrhea affects ? a- Cevix b- Urethra c- Poterior fornix of vagina The correct is A

26Qustion about fetal alcohol syndrome: a- Placenta inhibit the passage of alcohol b- Will cause fetla retardation and facial fetures and other symptoms c- It's safe to drink wine and hard something once a week Most likely B

278 year old boy weight and height above 95th centile what to do ? a- Surgery b- Medications c- Observation for 12 months d- Life style midification 28Abdominal pain mucus diarrhea .. no blood .. relief after defecation .. 3-4 times a day a- Ibs b- Ulcerative colitis c- A 29Acromegaly .. the cause a- Somatedin Correct

30Boy with presented with painless neck mass .. hx for 5 weeks of fatigue generalaize pruritis and mild cough .. dx

31abcde-

a- Hogkin's b- Lyme c- Infectious mono Maybe A

32In aseptic meningitis .. in the initial 24 hours what will happen? a- Decrease protien b- Increase glucose c- Lymphocytse d- Eosinophils e- Something Most likely C 33Patient with small congenital inguinal hernia a- It will cause infertility b- Surgey to be dona at 35 years c- Elective surgery if it's reducible

Fresh frozen plasma in what case ? Hemophilia a Hemophilia b Von willbrand Dic Coagulopathy form liver disease Maybe D

34Mass in the upper back .. with punctum and releasing white frothy material a- It's likely to be infected and Antibiotic must be given before anything b- Steroid will decrease its size c- It can be treated with cryotherapy

35Pt. after stroke or trauma I don't remember .. he lost his smell sensation.. which part is affected a- Frontal b- Temporal c- Occipital The correct B 36Doctor suspected meningitis in a pt. .. what's the immediate action for meningococcal a- Penicillin 1200 iv When meningococcal disease is suspected, treatment must be started immediately and should not be delayed while waiting for investigations. Treatment in primary care usually involves prompt intramuscular administration of benzylpenicillin SLE 26 nov 2011 Khaild Jebrin King Saud University 1- Ssri Pt after 2 month post MI cannot sleep what to give him

d- It must be removed as a whole to keep the dermis intact

a-zollibdon

b- diazepam a-

2- obese, HTN cardiac pt with hyperlipidemia, sedentary life style and unhealthy food HTN, obesity, low HDL

What are the 3 most correctable risk factor?

bcd3: abc4-

High TAG, unhealthy food, sedentary life High cholesterol, unhealthy food, sedentary life High cholesterol, HTN, obesity Pt with active hepatitis what medication should not to give Ranitidine Heparin Atrovastsin

Pt with 3 month hx of feeling like depression, recently

employ what you will do: a- Tricyclic bc- Psychotherapy support d- Moi 5-

6- Painful vision loss: a- Central vein thrombosis b- Central artery embolism c- Acute angle closure glaucoma

Epidemiology curve is: a- Plotting number of cases on time line b- Geographical places c- Case with similar diagnosis

7- Neonate with mass on his eye : a-Neuroblastoma b-Leukemia

8- 15 years old with palpitation and fatigue. Investigation showed RT ventricularhypertrophy, RT ventricular overload and right branch block what is the diagnosis : a- ASD b- VSD

c-

Coartaction of aorta

9- Young pt with vague central abdominal pain then shifted to RLQ with tenderness what is the diagnosis: a- Appendicitis b- Diverticulitis

10- Middle age pt come complaining of abdominal pain and he think he has gastric cancer he went to 6 gastroenterologist did 1 CT 1 barium enema and series of investigation all are normal what is the diagnosis: a- Hypochondriasis b- Conversion c- Somatization 11- 75 y/o female c/o hip pain after walking and busy day also prevent her from sleeping and continue in the morning for several hours what is the diagnosis: a- Osteoarthritis b- Rheumatoidarthritis c- Depression 12- 70 y/o male pt c/o knee pain after walking imaging showed narrow joint space with hypochondral sclrosing what is the diagnosis: a- Osteoarthritis b- Rheumatoidarthritis c- Reactive arthritis 13- Similar hx to Q withsimilar choices abHyderocholrathiazid Furosemide 14 pt with HTN on diuretic he developed painful big toe what kind of medication he was on:

16- Table with investigation Na 112 Osmolality 311 low What is the diagnosis? a- Connssyndrome b- Cushingsyndrome c- SIADH Diabetes insipidus d-

15- Pt with HTN on thiazide came to ER shouting from pain in LT big toe O/E the whole left leg is swollen and tender no fever what is the diagnosis: a- Cellulitis b- Gout attack

d- Tonsillectomy 19abc-

17- Repeated Q about nephrogenic Diabetes insipidus 18- A 5-year-old child came with earache on examination there is fluid in middle ear and adenoid hypertrophy. Beside adenoidectomy on management, which also you should do: a. Myringotomy b. Grommet tube insertion c. Mastidectomy What best explain coronary artery disease: Noatherosclerosis Fatty deposition with widening of artery Atherosclerosis with widening of artery

20-Mechanism of vitamin C in wound healing: a. Epithiliazation

a- Mildintermittent b- Mildpersistent c- Moderate d- Sever 23- about head and neck injury : a. Hoarsness of voice and stridor can occure with mid facial i njury b. Tracheostomies contraindicated c. Facial injury may cause upper air way injures 24- treatment of trigeminal neuralgia abcabCarbamazepine Phenotyn Phenobarbital

b. c. d. e. 21 ab-

22- pt with asthma on daily steroid inhaler and short acting B2 agonist what category:

Aerobic fibroblast synthesis Collagen synthesis Enhance vascularization ---- Pt on long term steroid what are the main complication Osteoporosis DVT

25- skin lesion pale pink on the leg of young boy not responding to 6 week miconazol what is the diagnosis: Tineacorpora Erythema nodusm

26- Mother worries about radiation from microwave if exposed to her child. What you tell her:

j. k. l. m. abc-

27- in pediatric what is the most common for failure to thrive :

Not all radiation are dangerous and microwave one of them Microwave is dangerous on children Microwave is dangerous on adult ---Protein& milk intolerance Psychosocial Cystic fibrosis i. ii. iii. gonnrhia Chalymide Trichmans

28- female with dysuria urine analysis showed ve gram diplococcic what is the organism :

iv. v. vi.

HBc HBs ag HBs antibody

29- Similar diagram about window period:

abcdabcd-

30-This diagram came the Q was typical onset of nephrotic in diabetes: 10 years 15 20 25

2 doses in 2 weeks 2 doses in 6 week 2 doses in 2 month 2 doses in 1 year

31 varicellavaccineyou will give:

abcdn. o. p. q.

Is not contraindicate in pregnancy Terminate pregnancy immediately Before get pregnant 3 months Its live attenuated BACTERIA

32- varicellavaccine in women wants to get pregnant

34- All of the following are live vaccine except: a. MMR b. Oral plio c. Varicella d. Hepatitis B vaccine e. BCG a. b. c. d. Achalasia Esophagitis GERD Diffuse esophageal spasm

33- there is outbreak of diphtheria and tetanus in community,regarding to pregnant woman: Contraindication to give DT vaccine Ifexposed, terminate pregnancy immediately Ifexposed, terminate after 72 hour Give DT vaccine anyway

35- Patient with retrosternal chest pain, barium swallow show corkscrew appearance:

e.

36- newly married woman complain of no pregnancy for 3 month Withunprotective sexual intercourse: Try more

abc-

37- teacher with vomiting and jaundice and 2 of his student no blood contact what is the best investigation: Heb A IgG Heb A IgM Heb B

On microscopic examination of this greenish fluid show grampositivecocci in chains: f. Streptococcal gangrene g. Chlostrideal gangrene h. Fourniers gangrene i. Meningocemia 39- old lady with skin changes near areola according to her because new detergent she used, if it didnt resolve after 2 weeks of steroid cream what you willdo: Mammography Cbc US

02-83 year old male had been stabbed on midtriceps, .

a- insulin something b- dehydration something

40- pt with sever vomiting and diarrhea in ER when he stand he feel dizziness. supineBp 120/80 on sitting 80/40 . when asking him he answer with loss of sensorium what is most likely he has

abc-

abcj. k. l.

41 pt diagnosed with EBV and discharged a few days later he came to ER and when taking hx he become tachycardia and hypotensive what you will do: Fluid management Urgent abdomen CT IV antibiotic with fluid

abab-

Paralysis Ongrowing mass (painless)(slow-growing) Pain Vaginal bleeding Contractions Amniocentesis Triple

42- most common symptoms of soft tissue sarcoma:

43- what is the dangerous symptom during pregnancy?

43- pregnant lady wants to know if her baby has Down syndrome what is the best investigation:

44- Patient came to you complaining of hearing voices, later he started to complain of thought gets into his mind and can be taken out a. SCZ b. Mood c. Mania d. Agoraphobia 45- Best sentence to describe specifity of screening test, is the population of people who: a. Are negative of disease, and test is negative b. Are positive of disease, and test is negative c. Are positive comparing to total other people d. Negative disease, positive test e. Positive disease, negative test

47- pt with recurrent gout what you will give him

46- mid clavcular fracture in neonate a- Must be intraoperative fixation b- Figure of 8 c- Non union is common d- Most of them heal by it self eababcabcAllopurinol Prbencide

48- pt with hypothenar muscle atrophy numbness on little finger EMG showed ulnar entrapment what you would do Physiotherapy Observation Surgicalrelease

49-newborn with eye infection

50-your advice to prevent plaque disease is: a-hand washing b-rodent eradication c-spry insect side 51-female with inflammatory acne not responding to doxycycline and topical vitA .want to use oral vit A what you should tell her: It cause birth defect

Oral antibiotic Oral steroid Topicalantibiotic

am.

53- child with barking cough and fever 38

abcdaba-

52- infant 48hs in ICU with jaundice mother healthy with previous term pregnancy what is the most likely the cause Sickle cell diseas Thalassemia Maternal fetal blood mismatch Hereditary genetic disease Croup Epiglottitis

54- what is the drug that comparable to laparoscopy in ectopic pregnancy? Methotrexate

56- Major depression management: a. Intial therapy even severs

55- Mother who is breast feeding and she want to take MMR vaccine what is your advice: a. can be given safely during lactation b. contain live bacteria that will be transmitted to the baby c. stop breast feeding for 72 hrs after taking the vaccine

57 15 y/o boy with +ve occult blood in stool .what is the best investigation: a. isotope b. barium 58-56y old present with vasomotor rhinitis a. Local anti histamine b. Local decongestion c. Local steroid d. Systemic antibiotic

59- Patient after accident, the left ribcage move inward during inspiration and outward during expiration: a. Flial chest a. DM

60-Greatest risk of stroke: b. Elevated blood pressure e. Smooking

d. Hyperlipedemia

c. Family history of stroke

61- Child with large periorbitalhemangioma, if this hemangioma causes obstruction to vision, when will be permenant decrease in visual acuity a. After obstruction by one day b. by 1 week d. By 6 months

c. by 3 months aCT

62- pt with renal stone what is the best invesatgation

63- increase survival in COPD ababO2 supplementation Smoking cessation

64- cause of non-traumatic subarachnoid hemorrhage Middlemeningeal artery Bridging vein

65- what's the organism responsible for psuedomembranouscolitis: a. b. c. d. Pseudomonas Colisteridum E.coli Enterococcus fecalis

66- another Q as above

67-18 months old came with bite by her brother, what you will do: a. Give augminten b. Give titunus toxoid c. Suture 68- what cause epistaxis in children: aSelf induced

69- pt on anti Tb medication with hear loss what is the cause: A-pyrenzmaid ba. b. c. d. 1300 1400 1500 1600 Streptomycin

70- you r supposed to keep a child NPO he's 25 kgs, how much you will give:

71- Deep jaundice wit palpable gallbladder?

a. Cancer head of pancreas

72- Lactating mother newly diagnosed with epilepsy , taking for it phenobarbital you advice is: b. Breastfeed baby after 8 hours of the medication a. Discontinue breastfeeding immediately

73-Pregnant women has fibroid with of the following is True: a. Presented with severe anemia b. Likely to regress after Pregnancy c. Surgery immediately d. Presented with Antepartum He

c . Continoue breastfeeding as tolerated

74- Patient with GERD has barretesophagus , this metaplasia increase risk of :

75-In holding breath holding which of the following True: a. Mostly occurs between age of 5 and 10 b. Increase Risk of epilepsy c. A known precipitant cause of generalized convulsion d. Diazepam may decrease the attack

a. Adenocarcinoma b. Squmaou cell carcinoma

76-All can cause gastric ulcer except: a- Tricyclic antidepressant. b- Delay gastric emptying. c- Sepsis. d- Salicylates. e- Gastric outlet incompetent

77-about specifity: a. test is +ve in disese population b. test is +ve in healthy poplation c. test is -ve in diseased population d. test is -ve in healthy population

78- Patient with high output fistula , for which TPN was ordered a few weeks ,2 unit of blood given and after 2 hours , the patient become comatose and unresponsive , what is the most likely cause : A. B. C. D. E. Septic shock . Electrolytes imbalance . Delayed response of blood mismatch . Hypoglycemia . Hypernatremia

A. Main complication is hypoglycemia . B. Can lead to weight gain . C. It suppress the hepatic glauconeogenesis .

79- Which of the following is true regarding metformin :

80- Patient with nausea, vomiting, and diarrhea developed postural hypotension. Fluid deficit is: a) Intracellular b)Extracellular c) Interstitial d)Vulva e) Cervix f) Urethra g) Ureter

81- Female patient did urine analysis shows epithelial cells in urine, it comes from:

82- Child patient after swimming in pool came complaining of right ear tenderness on examination patient has external auditory canal redness, tender, and discharge the management is: h)Antibiotics drops i) Systemic antibiotics j) Steroid drops

a- Pregnant woman should consume an average calorie 300-500 per day b- Regardless her BMI or body weight she should gain from 1.5 3 lb which represent the baby's growth a. verifying diagnosis

83- regarding weight gain in pregnancy what is true :

85-most common site of malignancy in paranasalsinuses : a. Maxillary sinus a. frequency of contractions b. strength of contractions

84-) 1st step in epidemic study is :

86- best indicator for labor progress is :

c. descent of the presenting part

87-A patient with penetrating abdominal stab wound. Vitals are: HR 98, BP 140/80, RR 18. A part of omentum was protruding through the wound. What is the most appropriate next step: a. FAST Ultrasound b. DPL (Diagnostic peritoneal lavage) c. Explore the wound d. Arrange for a CT Scan e. Exploratory laparotomy

88-A female patient with history of cyclic abdominal pain, inability to conceive, heavy menses, and examination showed tenderness & nodularity in uterosacral ligaments. What is the diagnosis? 89-A burn patient is treated with Silver Sulfadiazine, the toxicity of this drug can cause: a) b) c) d) e) Lycosytosis Neutropenia Electrolyte disbalance Hypokalemia .. SLE1 b King Saud University a. Endometriosis

Patient with CML taking imatinib mesylate and odansetron for nausea and vomiting presented with tachycardia,fever Diphoresis and hyperreflexia Dx: _ imatinib toxicity _ odansetron toxicity _ neuroleptic malignant syndrome ??

Not true

P3 with hx of D,C after 2nd delivery complaining now of amenorrhea with high( FSH,LH) low estrogen..Dx: _ turner syndrome _ ovarin failure _ asherman syndrome

Waking up from sleep..cant talk, no fever, can cough, normal vocal cordsDx: Functional aphonia +ve leichman test: ACL injury

All are 1ry prevention of anemia except: _ iron fortified food in childhood

_ health education about food rish in iron

_ limitation of cow milk before 12 month of age _ genetic screening for hereditary anemia Methylergonevine is # in: Maternal HTN _ iron,folic acid supp. In pregnancy and postnatal??

OCP that causes hyperkalemia: Estradiol p

Estradiol d(cant remember) Sever blepheritis with rosacea..Rx: _ topical ABX

_ oral doxycyclin _ doxycyclin _ ceftriaxon

Lactational mastitis..Rx: _ ciprofloxacin _ gentamyecin _ cephalexin _ central line _ 100% O2

Patient with ARDS on ventilation developed pnemothorax..cause: _ -ve pressure ventilation

( note : answer is +ve pressure or lung injury ) _ dust&pollens _u/v light _ stress

Recurrent watery discharge of eye, pain, sensitivity to light..on exam.> inflammation,ulceration of eye..cause:

_ night accommodation _ topical steroids _ reassurance _ abx

2 month infant with white plenched papules in the face what to do:

Young suddenly develops ear pain, facial dropping..what to do: _ mostly will resolve spontaneously

_ 25% will have permenant paralysis _ no role of steroids _ salmonella _ H.pylori

40 y/o with mild epigastric pain and nausea for 6 months..endoscopy>loss of rugeal folds, biopsy> infiltration of B lymphocytes..treated with abx..cause: Ashmatic child taking beclomethason that mostly cause: _ increase activity _ intraocular HTN _ preeclampsia

_ growth retardation

16 wk pregnant not known to have illness before has high BP..DX: _ gestational HTN _ chronic HTN

Patient with continous seizures for 35 min. despite taking 20 mg Iv diazepam..what to do?? _ give 40 mg IV diazepam _ give IV phenytoin _ nothing _ steroid _ give IV Phenobarbital _ amphotericin B

Pt after swimming pool(clear Dx of otaitis externa) Rx:

_ ciprofloxacin drops _ joint deformity _ lung cavitations

Which of following favor Dx of SLE?? _ sever rayaniod phenomen _ cytoid body in retina _ bed rest is the best

In patient with rheumatoid arthritis: _ cold app. Over joint is good

_ exercise will decrease postinflammatory contractures Before vaginal delivery, obstetrician should rule out: _ cephalopelvic disproportion _ cord prolapsed??

Pt. with 1st and 2nd degree burn involving face and neck: During heart contraction,heart receive more blood by: _ coronary artery dilatation _ IVC dilatation _ pulmonary vein constriction

All choices with no hospital admission except one which I choosed as the burn involves the face

-_ -

1- Fick method in determining cardiac output ; 1 BP 2 o2 saturation in blood 2- PVC caused by;

a decrease o2 supplement to heart b increase co2 to heart A_ osteoporosis 3_ old pt with 2 years bone pain , lethargy , fatigue, wedding gait , came with table show high calcium and high phosphorus ; B_ osteomalacia C_ paget disease of bone

D_ metastases prostate cancer E_ paraneoplastic syndrome

4- child with inferior thigh swelling and pain but with normal movment of knee , no effusion on knee what the important thing to do ; A_blood culture b-ESR c_ASO titer

d-aspirate from knee joint d-plain film on thigh 5-propylthiouracil drug mechanism of action: 6thing ;

a-Indwelling foley cather drain b voiding cytctogram c- cystoscopy A_ digoxin

7- drug contraindication hypertrophic obstructive cardiomyopathy; B_ one of b-blocker 8- BPH pt with hypertension what to give; alpha blocker

9- cold utritcaria treatmemt .-_10- chronic use of vasoconstrictive result in ; A_ rhenoid phenomenon B_rhinits sicca C_vasomotor rhinitis

11- posterior hip dislocation : ( from reconstruction) A flexion , adduction b- flexion abduction c- extension, adduction

12- action of ocp : ( from reconstruction) A - inhibition of estrogen then ovulation B inhibition of prolactin then ovulation c- inhibition of mid cycle gonadotropin then ovulation A hyperthyroidism B hypothyroidism C euothyroid

13- thyroid cancer associated with : : ( from reconstruction)

14- young fall high absent sound in right side and resounce percution first thing to do oxygen mank : 1 oxygen mask 2 tube thoractomy

15 using scissors or nail cut which cause sharp pain at the base of the finger in matacarpophlyngeal joint and the finger become directed downward in (mean flexed DIPj) and cause pain when try to extend the finger.. 1-trigger finger 2-tendon nodule 3-dupetren contracure 4- mullet finger 16- itching scale in pack of knee . face and ant elbow : A scapis B eczema c- contact dermitis

17 mitral stenosis : B - systolic low pitch

A diastolic high pitch C- diastolic low pitch A doxycylene

18 chylmedia non pregent treatment :

19- randomized controlled trail : B both group treated simalary C open allocation

A systemic assiment predictably by particpint

20 difference between unstable and stable angina : A necrosis of heart muscle B - .. 21- mechanism of destruction of joint in RA : A swilling of synovial fluid c- . A - ..

B anti inflamtory cytokines attacking the joint

22- most common malignant parotid tumor in children:

() : pt. with 10 days history of MI discharged yestarday .. present - today with sudne pianful left lim by exam limb is cold and pale .. Dx ( a - DVT ( hx of bed ridden but no swilling or hottness ( b- arterial thrombosis (can be,, he has atherosclerosis sure it's correct % my answer ) I'M ) c- arterial embolism human bite to the hand .. greatest risk of infection in which - posetion ( a- dpendent (my answer (clenched ( correct answer - finger extended - obsruction b/c case) << diffuse abd. pain with hestory of sursegy - lf adhesions ),m next invX a- barium enema sure b- barium follow throuw ( my answer) not c and d other barium .. pt. with ARDS in hospitl .. he devolop tension pnemothorax - : what the caus i'm sure about positeve <<< .a- negative pressure ventilation !!!!!!!!!! pressure .. but negative b- 100% o2 c and d were irrelevent there is pt. with PID there is lower abd. tenderness.. on pevic exam - small mass in xxxxxx ligamente.. Rx a- colpotomy 100% sure b- laprotomy c- laproscopy ..infertile pt. with 3 previous d/c .. otherwise healthy .. Dx - a- PCOS b- ........ syndrome in ob/gyn section ) c- Txxxxxxxxx syndrome. 100% sure (i got 10/10 (: :) remmember it is start with a T d- ....... syndrome e- shehann syndrome diagnosis is pregnant with heptitis .. best blood test to confirm the - : a- alkaline phosphatase

SLE,my new MCQs Exam Tue 20-9-2011

b- wbc c- STOG d- ESR it's TB outbreak ..and one pt. come to doing tubercalin test and - negative .. what to do a- BCG b- isonized all of the following is extrapyramidal Sx exept - a- dyskinisia b- akathesia c- xxxxx esia d- clonic - tonic convulsion months,, what is female come to with 3 UTIs history in last - your advise defecation a- wipe frome behinde to front after .b- take a bath insted of shaower c- increse flude intake in newborn exam .. what is more dangerous - a- hydrocele b- abcent femoral pulses c- CHD (d- breast with milk discharge (normal it's witch's milk .. gestation in NICU 900 gram wighet .. otherwise normal - <<<< what to do a- give hem milk orally b- glucose infusion c- broad spectren antibiotic ..healthy child with pRBC in urin 15 cells/hpf .. what to do - tricky quastion . a- repeat urin analysis for blood and proten having growth spurt ...Dx tebial tubercle pain .. in 13 y/o boy - <<<< .. ( ) a- osgood fracture b- strees fracture ......... -c old dibetic man with sudden unilateral visual loss .. thare is - .. multible pigmentation in retena with macular edema .. Dx ( a- retenal detachment ( wrong .. come with floters ( b- retinal artry occlosion ( wrong .. no chirry red spots ( fit with all data givine c- retinal vien thrombosis ( my answer .. it's

( d- dibetic retinopathy ( no macular edema y/o female become deaf suddenly.. her mother become deaf - :when she was 30.. Dx a- otosclerosis 100% sure b- acostic neuroma c- tympanic perforation ( all true) <<<<<<<<<< :major hazard in post-menepause - sure i'm 90% << a- ostioprosis b- hot flush c- deppresion d- pelvic floor weakness : define epidemlogy- .deases the scince of distrebution and determnant of : an alpha blocking effect which of the folloing b- blocker .. have - <<<<<<< a- metoprlol b- atenalol c- mesoprolol d- xxxxxx lol e- yyyyyy lol : miliary TB caractarized by - a- spare lung apical (b- septal line (not sure c- multible lung nudules to recognize primary snoring fros Questionnaire Screening the - : .. OSAS is Questionnaire a- otowa Questionnaire Horchover -b ......... -c ........ -d ADHD : <<<<< ADHD Rx - a- olanzpin b- atomixtin -c : epidemic curve is - read about it :case control study is - read about it cuz they will give very long defintion patient with congestive heart failure , which medicaion will - decrese his mortality

forsumide- digoxin - ....... - patient with congistive heart failure and pulmonary edema - :what is the best treatment spronalctone - .forsumide - .... post partum women when she went back to work ,, she exposed - the sun and started to have brown discolortion in her face .. what to :diagnosis is the uritcariA pigementosa?? not sure - : regarding hiatal hernia- (sympotmes increase with pregnacy (i choosed it - sympotmes increased with lying down- :child took 20 pills of paracetamol .. what u will give- ( (: N-acetylcystine (correct :regarding murmer of mitral stenosis - late distolic i think this is correct- holosystolic - mid systolic - new child moved with his family to new city n he started to go to- scool .. in the school he had low mood n doesn't want to interactive :any activity .. this a cas of with hypomania ! i think it is not correct - depression- palapitation ,, years old girl failed in math exam .. then she had - : tachypnea and paracethesia .. this is hyperventilation syndrome? i think this is the answer - conversion- : patient with IHD ,, best exercise is- areobic- :whic one will reduce colon cancer- vit d - zenc- !! no fiber no vit c :symptomes of open angle glacome- child with syomptomes of intestinal obstruction .. what- investigation u will do barium enema -

barium follow throgh - -----------------------------GD LUCK

SLE Exam dec 2011

() what is the most common malignant parotid tumor in children: a) b) c) d) a) b) c) d) a) b) c) d) e) a) b) c) d) Mucoepiptheloild carcinoma Adenocarcinoma Undifferentiated CA Undifferentiated sarcoma Cervicitis PID Cystitis Pyelonephritis

2- Female with dysurea and cervical motion tenderness:

3- What is the best diagnostic test for maxillary sinusitis: CT scan X ray Torch examination MRI Us IV lorazepam IV phenobarbital IV phynetoin IV haloperidol

4- A man is brought to the ER after having seizure for more than 30 min the most initial drug you will start with:

5- A 40 year old man who become sweaty with palpitation before giving a speech in public otherwise he does very good at his job, he is having: a) b) c) d) a) b) c) d) generalizes anxiety disorder performance anxiety agoraphobia depression floxitine diazepam halperidol amytriptaline

6- A women who lost her husband 2 weeks ago she is unable to sleep at all you will give her:

7- A diabetic pregnant with HX of fetal demise .. now is having a fetus who is healthy and her DM is very well controlled .. you will allow her for: 8- A picture of mid line swelling that moves with degilution: a) b) c) d) Colloid goitre Thyroglossal cyst Cystic hygroma Thyroid carcinoma a) C/S at 38 weeks b) induction at 36 weeks c) allow SVD

9- a man who is having a severe pain on his big toe with knee pain and examination revealed negative perferingent crystals:

a) uric acid deposit secondary to synovial fluid over saturation b) Ca pyrophosphate secondary to synovial fluid over saturation

10- Secondary dysmenorrhea is: 11- Patient with symptoms of blephritis and acne rosacea the best Rx is: 12- Female with dysurea, urgency and small amount of urine passed .. she received several courses of AB over the last months but no improvement .. all investigations done urine analysis and culture with cbc are normal .. you should consider: a) b) c) d) a) b) c) d) interstitial cystitis DM Cervical erosion Candida albicans a) Doxacyclin b) Erythromycin c) Cephtriaxone a) due to anovulation. b) due to gonadal agenesis c) always pathological

13- An outbreak of TB as a prophylaxis you should give : Give BCG vaccine Rifampicine Tetracycline H. influenza vaccine

14- A boy who was bitten by his brother .. and received tetanus shot 6 month ago and his laceration was 1 cm and you cleaned his wound next you will: a) b) c) d) give augmentin suture the wound give tetanus shot send home with close observation and return in 48 hr

15- in aspirin overdose:

16- A 42 year old man with cushing syndrome and had a fracture .. you should investigate 17- The specificity is: a) b) c) d) a) osteomylitis b) osteoarthritis c) osteoporosis

a) liver enzyme will peak within 3-4 hr b) first signs include peripheral neuropathy and loss of reflexes c) 150 mg/kg of aspirin will not result in aspirin toxicity

18- The important risk factor for Stroke is: a) DM b) HTN c) Dyslipidemia a) b) c) d) a) b) c) d) CPK ALP AST Amylase

When the person does have the disease with +ve test When the person does have the disease with -ve test When the person does not have the disease with +ve test When the person does not have the disease with -ve test

19- a man who has had MI you will follow the next enzyme

20- an old man who had stable angina the following is correct except: angina will last less than 10 min occur on exertion no enzymes will be elevated will be associated with loss of consciousness

21- Which of the following is given as prophylactic ant arrhythmic after MI: a) b) c) d) e) Procainamide Lidocaine Qindine Quanin Metoprolo

22- 5 yr old adopted child their recently parents brought him to you with white nasal discharge. He is known case of SCA. What you will do to him: 23- The antibiotic prophylaxis for endocarditis is: b) c) d) e) 2 g amoxicillin before procedure 1 h 1 g amoxicillin after procedure 2 g clindamycine before procedure 1 h 1 g clindamycine after procedure a) Give prophylactic penicillin

24- 19 yr old girl with URTI and splenomegaly. The cause: 25- Child with leukemia he has septicemia from the venous line the organism is: 26surgery the discharge is greenish showed Gram +ve cocci the organism is: a) Clostridium b) Streptococcus c) Pseudomonas a) E coli b) GBS c) Pseudomonas a) Infectious mononucleosis b) Streptococcus pharyngitis c) Malaria

27- a man travelled to Indonesia and had rice and cold water and ice cream .. he is now having severe watery diarrhea and severely dehydrated .. most likely he has: a) b) c) d) e) vibrio cholerae C difficele C perferngins Dysentry Shigella

28- When showed you refer a pt with scoliosis: 29- What is the best way of health education: 30- Pt with bilateral knee pain without signs of inflammation: 31- a man who is been in an accident just arrived at the ER you will: 32- an old man 65 years with Hemoglobin 9 .. you will: a) Assess Iron levels b) Assess LDH c) Arrange for endoscopy a) assess airway b) assess GCS c) Establish IV lines a) RA b) OA c) Septic arthritis a) Mass media b) Internal talk c) Individual approach a) 10 degree b) 15 degree c) 20 degree

33- computer programmer presented with wrist pain and +ve tinnel test. The splint should be applies in: 34- The most common cause of non traumatic subarachnoid hemorrhage is: 35- ECT is good for those: a) Middle meningeal artery hemorrhage b) Bridging vein hemorrhage c) Rupture of previously present aneurysm a) severe agoraphobia b) Severe major depression a) dorsiflexion position b) palmarflexion position c) extension position

36- most commo cause of otorrhea 37- A man who is having severe vomiting and diarrhea and now developed leg cramps after receiving 3 liters of dextrose .. he is having: a) b) c) d) hypokalemia hyponatremia hyperkalemia hypernatremia c) acute otitis media d) cholesteatoma e) Eustachian tube dysfunction

38- A man who bought a cat and now developed watery discharge from his eyes he is having: a) Allergic conjuctivitis b) Atopic dermatitis

39- Facial injury suturing remove after? a) b) c) d) a) b) c) d) a) b) c) d) 24h 3 5 days ( most likely ) 7 10 days 14 days Metoprolol Something with lol Acetazolamide Steroids

c) cat scratch disease

40- Acute angle glaucoma , with COPD and DM you give?

41- Child 9 month hx of congenital heart disease .. central and peripheral cyanosis Dx? Tetralogy of fallot Coarctation of aorta Truncus arteriosus ASD

42-Buprion use to quit smoking most common side effect is ? a) Seizure b) cough c) weight loss

43- And old man with low hemoglobin and IDA he is most likely having: 44- A female with dysurea invx showed presence of epithelial cells a) chlamydia urthitis b) cervicitis a) Hemorrhoids b) colorectal cancer c) ulcerative ****** not cloitis

45- a 3 year old with low hemoglobing eats lots of milk and very little red meat you will give : 46- Old pt complaining of back pain on walking on examination there was stiffness of the muscle and there was some finding on the X-Ray of spondyloarthropathy best effective ttt: 47- A child with congestive heart failure and several hemangioma on the body .. the most likely place for the hemangioma is: a) b) c) d) liver spleen intestine pancreas a) Physiotherapy b) NSAID c) Surgery a) trial of iron therapy b) Iron and multivitamin c) Send home with observation

48- Old pt with knee pain increase with walking , crepitus and stiffness for several hrs on waking Dx? 49- Newborn with clavicle fx: a) Osteoarthritis b) Rheumatoid arthritis a) Mostly brachial injury occur b) Mostly heal without complication c) d) Occur in the premature infant commonly a) obstructive b) inflammatory

50- a patient with acne of several appearances open .. closed .. red .. it is most likely:

51- a patient with a large nodule in the nose which is painful and talangectasia on the face you will give: 52- a man who received blood transfusion back in 1975 developed jaundice most likely has: a) b) c) d) e) a) b) c) d) a) b) c) d) a) b) c) d) Hep A Hep C Hep D Hep E Autoimmune hep a) deoxycycline b) clindamycin c) retenoid

53- A woman wants to take MMR she is breast feeding you tell her: may be given in breast feeding it contains live virus which will be transmitted to the baby contraindicated in pregnancy stop breast feeding for 72 hrs Hand wash Kill rodent spray pesticide give prophylactic AB Vit A Fluoride Zinc Calcium

54- Best method to prevent plague is:

55- Dental caries to prevent it .. mix the water with:

56- a man with high fever, petechial rash and CSF decrease glucose .. he has: a) N menengditis b) N gonorrhea

57- High senstive & specific for urolithasis the man had severe pain for one day and you suspect kidney stones : a) b) c) d) e) CT scan X ray MRI IVP US

c) H influenzae

58- 17 years old she missed her second dose of varecila vaccine,the a) give her double dose vaccine b) give her the second dose only c) see if she has antibody and act accordingly a) systemic study b) non randomized study c) stratified study a) anaphase b) metaphase c) telophase

59- In a certine study they are selecting the 10th family in each group,ahat is the type of study: 60- the separation of chromatid occur in: 61- a patient old with WBC 17000 and left iliac fossa tenderness and fever most likely has: 62- a DM HTN patient with MI receiving metformin and diltiazem and other medication his creatine clearance is high .. you will do: a) add ACE II inhbitor a) diverticulistis b) colon cancer c) crohn disease

63 - 70 yr old presented with wt loss, fatigue, anemia , upper quadtrant pain without any previous history, the stool sowed high fat he is a known somker: 64- a man after defecation finds blood on toilet paper he been having difficulties with defecation: 65- a baby with blood in the stool and bought of crying and x ray shows obstructive pattern.. looks like intosusseption you will do: a) b) c) d) a) b) c) d) a) b) c) d) surgery Barium enema observation giv e IV fluids and let obstruction solve itself call for neurlogist send home with close observation CT scan MRI 25% 50% 1% 4% a) colon cancer b) hemorrhoids c) anal fissure a) Acute pancreatitis b) Chronic pancreatitis c) Pancreatic carcinoma

b) remove metformin c) continue same medication

66- a child 3 years old fell from the bed vomited twice and has mild headache and no loss of consiousness .. you will:

67- Mother has baby with cleft palate and asks you what is the chance of having a second baby with cleft palate or cleft lip:

68- over oxygenation with 100% O will not result in: a) depression a) b) c) d) sublingual oral IM IV

69- Which of the following with antipsychotic medication have rapid onset of action?

70- which of the following contraceptive method is contraindicated in lactation: 71- the following is not a risk factor for coronary heart disease: a) b) c) d) a) b) c) d) a) b) c) d) High HDL HTN DM Hypercholestrolemia a) OCP b) Progesterone only c) IUCD

72- pregnant lady 16 wks presented with vaginal bleeding ,enlarged abdomen, vomiting ,her uterus is smaller than expected for the gestational snow storm appearance on US: Complete hydatiform mole Partial hydatiform mole Endometriosis Fibroids Mg sulphate Diazepam Phenytoin Phenobarbital

73 - the drug which is used in seizures of eclamptic origin ( pre eclampsia )

74- Female presented with thirst and polyurea.. all medical history is negative and she is not know to have medical issues.. .she gave history of being diagnosed as Bipolar and on Lithium but her Cr and BUN is normal. What is the cause of her presentation 75- child swallowing battery in the esophagus management : a) b) c) d) bronchoscope insert foley catheter observation 12hrs ( my answer) Remove by endoscope a) Nephrogenic DI b) Central DI

76- Strongest method to prevent the disease: a) immunization b) change health behavior of PPls a) b) c) d) warfarin heparin aspirin enoxparin

77- a female pregnant previously she have DVT you will now give her:

78- ibuprofen is contraindicated in: a) peptic ulcer b) seizures

79- In holding breath holding which of the following True: a) b) c) d) Mostly occurs between age of 5 and 10 Increase Risk of epilepsy A known precipitant cause of generalized convulsion Diazepam may decrease the attack

80- the best to give for DVT patients initially which is cost effective: a) b) c) d) a) b) c) d) LMWH Unfractioned Heparin Heparin Warfarin

81- Facial nerve when it exits the tempromandibular joint and enter parotid gland it passes: Superficial to retromandibular vein and ext. carotid artery deep to ex. Carotid deep to R vein between retrmandibular vein and external carotid artery

82- In hiatal hernia: b) opiates c) cocaine d) ectasy a) b) c) d) a) b) c) d)

83- a patient come to ER with constricted pupil and respiratory compromise you will suspect: 84- the treatment of trichomonas vaginalis: mteronidazole deoxycycline Ciprofloxacin Amoxacillin 40% 60% 80% 20%

a) It will increase with pregnancy

85another 30 develop same condition in the next 2 week , what is the attack rate

86- Contraindication of breast feeding: a) b) c) d) a) b) c) d) a) b) c) d) a) b) c) d) a) b) c) d) Asymptomatic HIV Active hepatitis C Veneral wart TB treated for 3 months

87- Case about a child with drooling, fever, barking cough in sitting position, dx: Croup ( acute treacheolaryngiobronchitis ) Broncholities Pneumonia Acute epiglotitis Imipramine and vasssopressine clonodine and vassopressine clonodine and guanfacine Imipramine and guafacine Somatization disorder hypchondraisis generalized anxiety disorder depression mulluscon contagisom viral wrats erythema nodusm chicken pox

88- a boy with nocturnal enuresis .. psychotherapy failed to show result you will start with:

89- a patient who thinks that he has a brain tumor with a long list of symptoms:

90- a picture of raised skin with black dot in the middle

91- Assymptomatic woman with trichomonas: a) b) c) d) e) a) b) c) d) a) b) c) d)

92- Pregnant women has fibroid with of the following is True: Presented with severe anemia Likely to regress after Pregnancy Surgery immediately Presented with Antepartum Hemorrhage repeated elbow trauma Autoimmune disease Staph. Aureus rupture of bursa

Treat if symptomatic Treat if she is pregnant Treat her anyway tell her to come in one month if she developed symptoms follow up

93- Olecranon Bursitis of the elbow joint caused by:

94- an old patient with the following labs Na was low and plasma osmolality or urine was low I don't recall it: 95- Which of the following is an indication for tonsillectomy? a) b) c) d) Sleep apnea Asymptomatic large tonsils Peripharygeal abscess Retropharyngeal abscess a) Cushing syndrome b) Addison syndrome c) Conn syndrome

96- Which of the following features is related to crohns disease: a) Fistula formation b) Superficial layer involvement

SLE Exam dec 2011 () -1REGARDING ECG:

P MITRAL .. P TALLER THAN 2.5 Q WAVE . I CAN'T REMEMBER

-2LBBB CAME WITH BROUDINING OF QRS MORE THAN 10 SEC -3TTT OF SCABIES :PERMETHIN

4-CHILD SEVERLY ILL AND FEVER FOR 2 DAYS ANOREXIA NAUSEA VOMITING THEN PETECHIA RASH APEAR IN TRUNK AND SPREAD IN THE BODY ?? MEASELS- MENINGOCOCCAL MENINGITIS MOUNTAIN FEVER 5-WHICH DRUG WE GIVE WITH ANALGISIC TO PREVENT S.E ? CIMITIDINE METOCLOPRAMID CT BARIUM ENEMA-COLONOSCOPY-SIGMOIDOSCOPY 7- RECOGNISED FEATURE OF HIATUS HERNIA ? -6BEST DIAGNOSTIC IN ACUTE DIVERTICULITIS ?

ANOREXIA MORNING VOMTING INCREASE WITH PREGNANCYLEUCOPINIA- SKIN PIGMENTATION 8-PT CAME WITH RAPID BREATHING ACETON SMELL GLUCOSE 500 ? HYPOGLYCEMIAL CASE LACTIC ACIDOSIS AND M.ACIDOSIS STARVATION BREAK PROTEIN ANF FORM KETONS 9- ANEMIA OF CHRONIC DISEASE ? UNCONTROLLED HYPERGLYCEMIAL CASE L.ACIDOSIS AND METABOLIC ACIDOSIS

10?RECURRENT VERTIGO-TINNITUS HEARING LOSS ? NUMBNESS IN 5TH FINGER 11 SURGICAL DECOMPRESSION CAT SCAN STEROID AND ANTIINFLAMMATORY

MEINERES CHOLESTEATOMA VESTIBULAR NEURITIS

CHEXEN BOX SCABES-ATROFIC DERMATITIS REFER TO OPHTHALMOLOGIST DIET RESTRICTION

ON RETINA EXAMINATION 31 02 NEOVASCULARIZATION 14 PT WITH SEASONAL NASAL IRRITATION AND SNEEZING ?PROPHYLAXIS ANTIHISTAMINE-STEROID-DECONGESTED 15- DRUG USED IN ATTENTION DEFICIT NITROFUNTON-CEPHLAXIN-SMT 16- PT WITH UTI ALLERGIC TO SULFA AND PENICILLIN ?

WRIST WITH PAPULOVESICULAR RASH 21

17- PT HAS ASTHMATIC ATTAK WITH EXERSISE AND COLD ?INITIAL DRUG? SHORT B 2 AGONIST STEROID INHALER THIOPHYLINE 18- PT WITH MENINGO MENINGITIS DRUD OF CHOICE ? PENICILLINE DOXACILIN

19- LENGHTH OF SPINAL CORD AFTER MENARCHE CONTINUE FOR ? 6 M-12M-24M-36M

20- RAYNOD SYNDROME PASSIVE SMOKER BLUISH HAND AT COLD WHAT U WILL ADVICE HER ? PASSIVE SMOKER DOES NOT AFFECT HER USE ANTI VIBRILATION GLOVES WARM HER CORE BODY TEMP AT COLD DAYS

21-GOLD STANDERS TEST IN RENAL FUNCTION ?

CREATININE CLEARANCE 24H CREATIN- ??RATIO

HIV 22 !!!! PNUMOCYCTIC CARNII-KAPOSI SARCOMA BLADDER CANCER PROSTATE ENLARGMENT- PELVIC CANCERFIBROSIS ADENOMYOSIS-ENDOMETRIOSIS-UTERINE LIOMATA-UTERINE CARCINOMA

23-PT WITH LOIN PAIN ON US HYDRONEPHROSIS OF BOTH URETER ? CAUSE ? 24- PT DYSMENORRIA DYSPARUNIA INVERTED UTERUS ??????? 25-which drug C/I in case of obstructive hypertrophic cardiomyopathy ?

B blocker ASA Digoxin 26-Child with mild Truman develop hemoarthrosis , in past hx similar episode Dx THROMPOCYTOPENIA- FACTOR 8 DIDICIENCY-

27- PT GO TO PHC FOR HIS MIGRANE MEDICATION HE RECENTLY DEVELOP EASY FLUSHING OF THE FACE AND LARGE NOSE !!

BEFORE 2 DAYS OPHTHALMOLOGIST DISCRIBE TO HIM FOR KERATITIS AND BLEPHARITIS WHAT IS THE DIAGNOSIS??? ATOPIC DERMATITIS ROSACEA.. 28-1RY OSTEOPROSIS CASED BY ? CA INTAKE VIT D INSUFFICINCY-AGING PROCESS-EXERCISE 29- PT HAS COLON CANCER STAGE C2 ROLE OF SURGERY ? CURATIVE PALLIATIVE-EXPLORATORY-DIAGNOSTIC 30-PT IN HER 4TH DAY AFTER C SECTION WE FOUND HER PROFOUNDLY HYPOTENTION ?INITIAL ACTION? 31- RT LUNG : 1 FISSURE-2 PULMONARY VEIN 7 SEGMENT

GIVE 0.9 NS WITH NACL ALBUMIN- DO SEPTIC WORKUP AND START ABX

SLE Exam 2011 by :Omar Alharbi King Saud University

SLE Exam 2011 from: King Saud University

regarding anticaogulation therapy .. vit k reverses the action of warfarin IV ca++ channel blocker nitro paste iv morphine beta blocker

* long scenarion of MI , the q is ,, inappropriate management:

* chronic diarrhea is a feature of ... typical men malzamat alqaeem * ARDS , cause of pneumothorax, central line insertion damaged lung negative pressure vent.

* pt is taking tx for glaucoma, now having SOB and Cough .. what medication is he on ?

pilocarpine timiolol high Bp

* first indication of pre-eclampsia is abruption Pv bleed

* pt GA 37 week , painful pv bleed, dx ,,, abruption trachoma candiada

* dysuria + yellowish greenish discharge.. B vaginosis

* pt, GA 34 breach presentation , what to do? wait and reassess by 36 weeks * pt have cheesy pv material ... candia trachoma B. vaginosis

* photo of skin rash,

dx, sebbrhoic dermatitis

don't worry typical description

* graph , women and osteoprosis

can't remember the q , bas ka2annaha feeh fe malzamat alqaseem * red tympanic membrane, + hemorhagic vesicle ! ur organism is ... strept pyogenes H influenza * bluging tympanic membrane: tx amoxicillin * basic science regarding IHD , clozapine chrons UC

all answers are crappy except for, HTN is a known risk * antipsychotic , least to cause tardive dyskinesia .. * pt has diarhhia and occult blood and colonoscopy is showing friable mucosa , bipposy is showing g crypt abcess .... * pt with UC and tender nodule ... erythema nodosum * tx of pyodrma gangrenosum .... oral anbx

iv anbx

local anbx MAOI

* pt on antidepressant medication, having insomina and restlessness what is he on .... SSRI * pt has EBV, during abdomen exam., became pale with tender LUQ ..... IVF / Urgent CT rush him to OR staph * sutured triceps post trauma, greenish material gram +ve in chains strept

* MCC of physiologic hypoxemia .... don't know, don't care :P * type 1 DM ... HLA DR4 * scenario typical of CA esophagus * DM, tired and blood glucose is 60 ... he is on sulphonylurea

*doesn't go with head engagment ... 3/5 of head in abdomen

* urinalysis, epitheilal cells ..... contamination,same as alqaseem malzama scenario of hypothyroididm

* mother hep B positive , son is also positive .... vaccine ,,, same as alqaseem malzama SLE Exam 2011 Dr.Amani Alahamri King Khaled University

Patient complain of infertility obese hirtsuim acne . Ultrasound show .. multiple cyst What is most likly DX Polycystic ovary sydrom and ask patient to decrease the weight pateint complain of RUQ pain radiated to shoulder and fever ( -2 picture of acute cholycystits) what is most usful test in DX Ultrasound pateints with appendicits what is most helpful to make DX -3 Age Fever High WBC High sedmentation rate paient suspected to have connective tissue disease what is most -4 favurable to SLE Cystoid body in fundoscopy Cavitaion in lung ve anti RNP+ Sever Ryundoe phenomena y develop sorethorate on-5 examination there is congested thorat and pharynx and white to papule on erthymatus base in mouth and lip what is most yellowish

likly DX Coxsacki virus Herps simplix virus Child with high grade fever .. Cough drooling of saliva and Stridor -6 TTT () Paracetamol Admitted to ICU snd call the ENT psteint fall from ladder .. Breathing labored and pt cynosed on -7 examination silent chest over right lung with resonant on percussion What first step Chest x ray Cricothyrodtomy Tracheostomy Oxygen mask Oral endotracheal tube patient with breast cancer and metastasis came complain of -8 tachycardia hypotension , engorged neck vein and SOB what is most action next D-dimer Ventlation prefusion scan Give fursamide and refer to do echo 9- child came with fever and ear pain on examination ( the same -10 picture of otitis media) ttt Amoxacillin Cefruxime Which drug can not be use in acute cholysystits-11 Naproxen Morphine Mepriden Acetamenophin Perdoxyphen Which drug can use in acute back pain -12

Diazepam Alprozam Metoxelen old patient came with fever . left LQ pain and tenderness but no -13 abdominal distention Sigmoid volvuls Diverticulitis Intestinal obestraction DX abdominal pain and bloody diarrhea Ischemic colitis (pateint with decrease lipido and weak erection ( or ejaculation -15 In investigation prolactin high , LH and FSH normal what is next step Brain MRI Abdomen and pelvic CT all live vaccine except-16 Hepatits B ttt of non inflamatory acne -17 Clindamycin Retonic acid Isoretinon picture of bulls in food ... In biosy there is epidermal lysis and on -18 immunoflurescen: deposition of IgG DX Bulls pemphigoid Pemphigoid valgarius perthes disease all except -19 -14

Can be presented with painless limp It alawys unilateral ... 20-Athelet man came complain of pain in foot wiyh walking on excamination there is tenderness in planter of foot what is DX Planter faciitis Halux vagus Hallux rigidus ) ) pateint complain of left elbow pain -21 lateral epichondylitis relative risk 22- paient -23 dysmeanorhea) DX Endometriosis Pelvic congestion Endometritis and examinatin normal what is ttt Mafnemic Compined pill ... : withdrwal symptom of stop smoking -25 day 2-1 day 4-2 pateint came with cough ,SOB snd wheezing he had past hx of -26 and pelvic pain .all investigatin -24

glucoma what is most likly drug cause this Betoxelol Timolol prinetal mortality rate 72- ) patient with PPD test positive -28 Isonized and rifampcin for 6 month Isonized for 6 month Isonized and rifampcin and streptomycin for 12 month (: .. Considerd positive mantux test in .29

) Induration more than 10 mm in philpine man (: .... .. old patient C/o stifness in knee and bilatral enlargment in knee -30 without effusion ESR and CRP normal DX Osteoarthritis you want to give varcilla vaccine in one no have vaccin before -31 Two dose and 6 weak between pregnant not vaccinated against measls and mumps and rublla .. -32 She exposed to rublla 3 day ago what you do No treatment Immunoglobin Tell her no affected on her pregnancy if she take the vaccine

33- in -ludwig angina most common complication Asyphaxiation -43 patient with vertigo ... Progressive hearing loss. What you find in MRI Acustic neuroma 35- Pateint complian of diplopia , weakness , and frequant aspiration pnumonia in last 2 month ... In examination there is spascity and fasciculation DX Mythenia gravis Mythenia syndrom Motor neuron disease 36- child with low grade fever , sore thorat in examination there is lymph node enlarment but not tender and no exudate on phrynx DX It is most likly streptococcal than viral It is viral more than bactrial Most likly EBV 37- what is the. symptom Most likly occure with hiatus hernia Skin pigmentation The symptom increase with pregnancy ... << 38- patient with bilateral eye redness . Discharge and tearing on examination cornea , lens all normal Nd tere is conactival follicle DX Acute conjactivitis 39- child C/O fever , sore thorat all examination was normal What is

the ttt : Cefruxime Ceftriaxone Give paracetamol and take pharenx swap 40- pateint C/o ictrus in skin and eye on investigation WBC 2500 plt 70,000 HG 7 lekocytosis 17% total bilirubin 51 and direct bilrubin 12 what is the test most likly positve +ve coomb's test In US obestructive billiary duct jhemolysis 41- Child not complain of any thing all investigation and examination was normal except the pharengeal swap +ve for meningiococal ... What ttt No need for ttt IM one dose of ceftriaxone 42- non pharmalogical ttt of osteaoartritis Analgsic cream Strenthing muscle excerise 43- kawshirkor Low protin and high carbohydrate Low protien and low carbohydrate 44 ... Because the organsim develop resistant Develop new antigenic drift 45- unfavurable risk factor of schezophrenia Family hx

46- infant with high grade fever .. Irritable .. Look sick .. Complain of anuria 4 hour with multiple petechiea and purpura on body .. He was tachycardic and hypotensive DX Renal fauiler Septic shock 47-patient c/o low self steam and fatigue .. Lack of intersted and concentration loss of sleaping , depressed mood for last 2 years what DX Dysthymic 48- patient with bed sore involve skin and extend to fascia what a grade Grade1 Grade 2 Grade 3 Grade 4 49-lacteting mother complain of fever and breast tenderness and redness diagnosed as bactrial mastitis what is ttt : Continoue breastfeeding and hot compresser and antibiotic Discontinue breast feeding and give antibiotic to mother and baby 50- patient with rhumatic heart disease and had mitral valve stenosis Mitral vave diameter less than 1 mm In order to maintain COP what will happen : Left atrial hypertrophy and decrease pulmonary prusser Left atrial hypertrophy and champer dilitation RV hypertrophy and decrease pulmonary prusser RV hypertrophy and champer dilitation ...

// ... .. .. ... ...

....

In the name of allah My SLE exam (19-12-2011) 1back, insomnia, 8kg loss O/E there is mild jauindice, and he is t the criteria of depression what is the cause of his depression: MDD Dysthimic disorder Adjustment disorder depression secondary to medical condition

2every thing normal except decrease in visual acuity 20/100 in rt. Eye, 20/160 in lt. eye ,, cornea, lens, visual eld all within normal ,, on fundoscopy you find early cataract formation in both eye what you will do: Refer to cataract specialist for cataract surgery Refer to ,,,,,,,,,,,, ,,,,,,,,,,, ,,,,,, laser correction

Unergent referral to ophthalmologist

3-post-partumbreastfeeding lady complain of breast engorgement & tenderness what you will do: Warm compressor before feedinf Cold ,,,,,,,,,,,,,,,, ,,,,,,,,,, ,,,,,,,,,,,

They mention some antibiotics

4- pt. complaining of retro-sternal chest pain more in morning, after food when laying down..Dx: GERD

5-most common cause of epitasis in children: Polyp Self induced injury

6- typical scenario of closed angle glaucoma, Rx: IV ..+% eye drop Other choices not accepted (IM, oral)

7-child waking from sleep with crustations what is Dx; Bacterial conjunctivitis

8coming with new complain which is parasthesia & numbness in Lt. side of face what is the Dx: Aquistic neoroma Mennier disease Laberyinthitis

9-typical case of noncomplicated UTI, Dx: TM+. 3days

10- filling defect in renal pelvis not opaque, on US echo.. (they prescribe the apperence of this filling defect but I forget it) what is this filling defect: Uric acid stone Blood clot Epith. Cells Vascular 11- old patient you suspect cognitive dysfunction what to do before sending the pat. To home: Do brief IQ test Asses for hearing loss Click test (not sure for name) . Chart 12- true regarding syncope: Most common cause of fall in elderly If without warning sign it suspect vasovagal attack 13Hypothermia Hypoxemia Carbon monoxide poisoning Hypokalemia 14-Rx. Of scabies in pregnant women: 15-what is the mechanism of action of aspirin: Inhibit cyclooxygenase

Inhibit lipooxegenase 16- what is the drug that will preserve the histology in primary liver cirrhosis: I couldn`t remember the choices but one of them is interferon. 17-young lady every thing within normal regarding her menses but there is 7cm mass in ovary, what is it: Benign ovary teratoma Luteal mass Follicular mass 18- what is the most economic twice daily prescribe NSAID: They put a list of nonsteroidal!! one of them ibuoprophen!? 19- child on supplementation, coming with nausea, vomiting & diarrhea with black emesis, you suspect a toxicity of: May be iron because the stool is black

20- acute loss of body fluid in abdomen will cause: Sepsis Hypovolemic shock 21- contraindication of breast feeding: Asymptomatic HIV 22- Which one of the following need to add to water to decrease the incidence of dental caries: Fluride 23- all are risks for IHD except: High HDL

24- Strongest risk factor for stroke: HTN 25- Postpartum lady with post partum psychosis, which of the following is an important part in her management: Family support 26- case of tension headache (Dx.) 27- most common cause of nont-raumatic subarachnoid hem.: Rupture aneurysm 28- which of the following will indicate recurrent breast cancer: This Q. is repeated so you can search for the choices cuz I can`t remember them. 29- pat. Complain of occipital & neck pain DX: Cervical spond.. 30- true about UC: Increase risk of malignancy 31- true about crohn`s disease: Fistula formation 32- typical case of acute cholecystitis 33- role of surgery in stage C2 colon cancer: Curative, palliative, diagnostic , exploratory 34- epidemic curve: 35- pt. with exertional dyspnea, PND & mild pedal edema (not sure for Ch. HTN (no Hx. of HTN in the scenario!!)

Valvular heart disease ((no HF in the choices!!)) 36- child with enuresis which investigation is important: 37- indication of tonsillectomy: 38- best Rx. Of sleep apnea: I couldn`t remember the choices but one of them is surgery (vulvo. Surgery) 39- pt. with LBBB on ECG, otherwise normal regarding examination &echo, he will do dental procedure, what is your advice: Endocarditis prophylaxis before procedure ,,,,,,,,,,,,,,,,,,, ,,,,,,,,,,,,,,,,,, after ,,,,,,,,,,,,,,,, No need for prophylaxis 40- what is common CHD associated with endocarditis VSD ASD PDA Tetralogy 41till birth ,3 died within months ,2 died before their 1st birthday , with 750 p come out & 250 come in what is the birth mortality rate in this city : a. 4 b. 6 c. 8 d. 9 42-old pat. Complaining of abdominal pain , vomiting O/E there is long longitudinal scar in abdomen, on abdominal X-ray there is air fluid level, what is the next step:

Nasogastric decompression & IV fluid 43- what is the best investigation regarding renal function:

Serum creatinine level Inulin!! Level 44- 3w old neonate with projectile vomiting Dx. : Pyloic stenosis Duodenal atresia Hirschsprung disease 45- female with positive urine pregnancy test at home what next to do: Sreum beta HCG CBC 46- Pict. For zoster, with scenario asked about Rx: Acyclovir Famciclovir 47- other case of zoster (old pat. With back pain then develop vesicular eruption at the same area band like with tenderness) DX. 48- case of prenicious anemia 49- which true regarding pregnancy: Use of anti-thyroid medication increase the incidence of congenital anomaly GERD increase the incidence of IDA I forget the other choices 50- all are primary prevention of IDA except:

51- typical case on ankylosing spondylitis ask about Rx. : 52- old pat. With diffuse hair thinning & loss of eye lashes( he mention at the end of scenario that this pat. Admit to pull out his hair in stress) ask about Dx: Trichotillomania (the pattern of hair loss will not be diffuse) Other choices !!?? Vague Q!! 53- typical case of pertussis (ask about Dx.) 54- pat. On (name of drug may be diazepam ) complaining of muscle pain & spasm, also he complain of cubital spasm after removing b.p cuff the cause of his problem is : Hyponatrema Hpokalemia 55- old pat. With tremor , mask face. Dx: Parkinsonism 56- which of the following is true regarding alzahimer: Brain atrophy will be more in (he means not diffuse atrophy) Sulci widening more in frontal than occibital There is plaque 57- LONG case of SCA at the end he ask about what of the following is best to give: Penicillin 58- why flu vaccine need to give annually: Viral drift 59- the commonest presentation in abrupto placenta is:

Vaginal bleeding 60- 43y old female with irregular menses 3m back & 1next to do: US Human chorionic gonadotropin Placental ,,,,,,,,, ,,,,,,,,,,,,,,,,,,, FSH LH 61- newly married couple (6m ) not conceive what to do: 62- case of depressed man after death of his son, he can`t sleep at all for 2days, which drug will for short term: Lorazepam Imipramin 63- child already he is on supplementation, he develop nausea, vomiting & diarrhea. The emesis color is black , what toxicity he suffer from it: Iron 64- boutonniere deformity: PIP flexion with DIP hyperextension). 65- case of perth`s disease what is the appropriate management: Physiotherapy Surgery Non weight bearing for 6m 66- Patient is known case of cervical spondylolysis , presented by parasthesis of the little finger , with atrophy of the hypothenar muscles ,

EMG showed cubital tunnel compression of the ulnar nerve , what is your action now : cubital decompression . 67- true regarding trichomoniasis: Green , frothy discharge 68- Using the following classification : Risk factor Case (disease) Non case total Present A B A+b Absent C D C+d Total A+C B+D Relative risk of those with risk factor to those without risk factor is: r. A/A+B , C/C+D s. A/A+B t. C/C+D 69- null hypothesis: 70- Rx. Of trichomoniasis 71- adult pat. With mod. Persistant asthma on short acting bronchodilator & small dose inhaled steroid (the rest of scenario I didn`t understand it, but he mention that pat. Need to take drug twice daily!! Increase the dose of steroid inhaler Theiphylin + steroid + steroid 72- diabetic with arterial insufficiency (ask about Dx.) 73- typical case of OA 74- child with congested throught & tonsil with white plaque on erythematus base on tongue & lips , also there is gingivitis (Dx.)

75- case of epiglotitis ask about best next step regarding the management of this case: ICU with . other choices not appropriate for this emergency case 76- other case of epiglotitis ask abo 77- case of flail chest (ask about Dx.) 78- post. Pad sign 79- case of acute pancreatitis next step: Total parentral nutrition Jejuna nutrition 80- treatment of anaphylaxis: Epinephrine 81- Prolong use of vasoconstrictive nasal drop will cause: Vasomotor rhinitis Rebound phenomena

Good luck

This is my exam in saturday 24-12-2011 ( sorry i chouldn't remember all Q but these will help ) ---------------------------------------------------------1Q)difination of case control study ? 2Q)regarding prevention of plaque : rodent eridication 3Q-pregnant with HIV , the most accurate statment regarding risk of transmission of HIV to the baby : Placenta Through cord blood Contamination of the hands Breast feeding ?!!!! 4Q-pregnant with uterine fibroid , has no symptoms only abd. Pain , US showed live fetus ,,,,, What is the appropriate action to do: Myomectomy Hysteroectomy Pain management Pregnancy termination 5Q)10 months old baby came to the clinic with his mother , she breastfeed him 3 times a day ,, she is known cace of epilepsy on phenobarbital,,,,,, What u going to tell her : Stope breastfeeding immediately Weaning over 2 weeks period Breastfeed after 8 h from taking the drug Respond to what the mother and child wish 6Q-Elbow fx , on lateral xray : Post. Fat pad sign 7Q-Pt came with eye pain, watery discharge and light sinsitivity Eye examination showed corneal ulceration. Her symptoms are frequently repeated . Which of the folowing is triggring for recurrence of her symptoms: Dusts

Hypertension and hyperglycemia Dark and driving at night Ultraviolet light and stres( 100% sure ) 8Q- p.t taking a medication , came to the ER suspecting she has overdose of her medication, her symptoms ( convulsion, dilated pupil, hyperreflexia and strabismus) the medication is: TCA ( 100% sure ) SSRI Hypervitaminosis 9Q-Pt complain of hearing voices from the microwave and refrigerator Visual hallucination Auditory hallucination ( 100 % sure ) 10Q)Old retired man having ansomnia only . Has no symptoms related to anxity or depression .. U will give him : Diazepam ( sure 100%) 11Q-pt take cephalexin after tooth extraction for days After that he develop profusre , green foul smilling diarrhea with low grad fever . He has tachycardia and mild abdominal dist. Sigmoioscopy showed white mucosal patches , what is the most ttt for this condition? Clarythromycine Vancomycine Cephalosporine Lineozides 12Q-pt with COPD came with couph , wheezing and greenish sputum The causative organism: H.influanza Strep.pneumonia Chlaymedia Mycoplasma pneum. 13Q-what is the most effective measure to limting the complications in COPD: Pnumococcal vaccination Smoking cescation 14Q-pt with hypertrophic subaortic stenosis ,, want to do tooth extraction ,, regarding to development of endocarditis : High risk 50%

no need for prophylatcic antibiotics Post procedure antibiotics are sufficient Low risk 12% 15Q)25 years old female came complaining of difficult hearing , she mentioned that their a family history of early oncet hearing loss ( her grandmother) Oto. Exam was normal .. Weber and rinne tests result in ( bone conduction is greater than air conduction ) ... Next action is : Refer her for aid hearing Tell her there is no avalible ttt Refer her to otolaryngologist 16Q)Old man came complaing of progressive hearing loss , it is mostly profounded when he listining to the radio, he does not has any symptoms like that before Weber and rinne tests result in bilateral sensorineural hearig loss.. Diagnosis: meniere's disease Otoscelerosis Noise induced deffnese Hereditary hearing loss 17Q)Prophylactic antibiotics after appendectomy Cephatrixone Metronidazole 18Q)Pt came with cough , wheezing , his chest ascultation revealed monophonic sound , on xray ther is patchy shadows in the upper lobe+ low volum wirh fibrosis ,, he lives in a crowded place .. What is the injection shuold be given to the pateint's contacts : hemophe.influanza type b Immunoglobuline Menngioc. Conjugated C Basil calament .... !!? 19Q)The most common cause for chronic irrigular rectal bleeding is: Diverticulitis Hemorrohids Colon cancer UC 20Q)What is the term used to describe the increase of the frequency of

the menstrual cycle: Ammenorrhea Dysmenorrhea Menorratogia Hypetmenorrhea Polymenorrhea 21Q)45 years old female came to ER with acutely swollen knee + ballotment patella .. The most important to do is: MRI of the knee Aspiration Complete blood count Rhumatoid factor 22Q)Pt came with a history of about 12 dayes duration severly red , swollen painful first metatarsophalangeal joint.. He is hypertensive with inverted T wave on ECG ... The most appropriate meaure for diagnosis: CBC Uric acid level Troponin level C-reactive protein 23Q)Adult P.t recived a vaccine ( i don't remeber the name ) After that he complain of itching , tachycardia and SOB What is the ttt? IV hydrocortizone 500 mg SC epinephrine 24Q)Pregnant on iron supplementation throughout her pregnancy for her anemia , now she come complaining of weakness and easy fatigability Her Hemoglubin 7 , MCV 60 .... What is the diagnosis? Iron def. Anemia Hypothyrodism Vit B12 def. Beta thalassemia 25Q)Picture ( diagram related to BMI ... So clear ) With a scenario of a female with BMI 32.6 Normal Overweight Obese Morbid obesity

26Q)Picture ( xray for intestinal obstruction ) With very clear scenario and descreption .. The Q about what to do ? Remove the obstruction ilues management Intest. Decopression 27Q)Pt has a scaly hypopigmented macules on the chest and arms They seem even lighter under the sunlight,,, what is the ttt? Topical steroid Na selinum Topical antibiotics Oral antibiotics 28Q)Repeated Q about baby who can name 4 colors .... His Age : 48 months ( 4 years ) 29Q)Pt came with left lower quadrant pain + fever and vomiting On examination there is left lower quadrant tenderness with localized rebound WBC 17.000 .... What is most likely diagnosis? Diverticulitis Granulomatouse lesion of crohns Intestinal ischemia Sigmoid vulvolus 30Q)Pt came after fight ( gunshot ) there is a pice of the omentum coming out from the wound . Vital signs ( HR 98 , BP 130/80, RR 18 ) .. What is the best action to do ? CT DPL Fast us Wound exploration Scheduled laprotomy 31Q)Which of the following breast mass is bilateral : Paget disease Lobular carcinoma Mucinouse carcinoma 32Another Q about which breast mass present with bloody discharge ? ( i didn't remember the choices , sorry ) but u should read about it 33Q)Most Dangerouse sign during pregnancy?

Vaginal bleeding 34Q- recurrence if clef lip in next pregnancy : 4% 35Q- regarding perotinitis which is true : chemical erosion 36Q- Why influenza vaccine given annually : viral antigenic drift 37Q- ECT Indicated in : sever depression with psycho motor retardation 38Q- clear scenario about appendicitis 39Q- repeated Q regarding newborn clavicular fx 40Q- repeated Q one statement is true regarding head and neck injury 41Q) Child came with his parents to the clinic , their parents said that their son looks bigger than the other children on his same age His BMI 34 ... His w.t and h.t on the growth chart is greater than his age Your advice will be: Life style modification Decrease fat intake 42Q) repeated Q about the puberty of the females earlier than the males By 2-3 years earlier ( true ) 43Q) the most common cause of epistaxis in children is: Nasal polyps Self induced 44Q)female patient presented with migraine headache which is pulsatile, unilateral , increase with activity . Dosn't want to take daily medication. Which of the following is appropriate: Bio feedback Triptan BB CCB

45Q)old man with bilateral knee pain and tenderness that increase with walking and crepitation relieved by rest: OA 46Q)lady c/o headche bandlike pain tension headache 47Q) child came with generalized body swelling, fever , dark urine with decrease urine output ,,, what is the most useful investigation for diagnosis: CBC Renal function test Abd. US Urine sedmintation test 48Q) one of the folowing manifest. As croup: Forigne body Pneumonia Common cold Asthma 49Q) all live vaccine except : Hepatits B 50Q)COPD patient with emphysema has low oxygen prolonged chronic high CO2, the respiratory drive is maintained in this patient by: Hypoxemia 51Q) clear scenario of varicocele ( bag of worms scrotum ) 52Q) repeated Q about refering pt with scoliosis at which degree 20 53Q) relation of indirect inguinal hernia to the spermatic cord anteromedial 54Q) scenario of glucoma in old pt ,, what is the best ttt? Acetazolamide + pilocarpine ( sure 100% ) 55Q) clear scenario of keratitis .. on examination there is dendritic ulcer: Herpes simplex keratitis

-------------------------Good luck .. wish u all the best Rawan Alshreem :)

..

1- What is the true about obstructed labor : a- Increase with antroposterior position b- Related to polyhadramnes . 2- What is the true about antepartum hemorrhage : a- Indication to vaginal examination . 3- What is the correct about unstable angina : a- Same drug that use in stable angina . b4- What is the side effect of steroid on the eye ? a- Glaucoma . b- Cattarect . c- Keratoconus . 5- What is the main side effect of silver salazin in burn : a- Acidosis .

b- Skin discleration . 6- Scenario for COPD 7- Scenario for TB . 8- The antidote for organophospherous is : Atropine . 9- What is the true about appendicitis in elderly : 10The best investigation for acute diveticolitis is : a- US b- Barium enema . c- CT d- Colonscopy e- Sigmidscopy 11Degree of scilosis that referral to orthopedic clinic : 20 12In infant with bleeding problem give him : Vit K 13Defenciy of B1 called : BeriBeri 14In DM : a- DR4 b- DR5 c- DR7 d- DR8 15Pap smear 16Patient with history of AF + MI , the best prevetion for stroke is : ? a- Warfirin b- Surgery procedure c- Shunt 17Profeational player came with history of truma on the lateral side of left knee , on examination there is swelling in the medial aspect of left knee , the diagnosis is : a- Medial collateral ligament spasm . b- Lateral collateral ligament spasm . c- Medial meniscus tear . d- Lateral meniscus tear 18Food poisoning , group of people came with diarrhea and vomiting diagnosis is: a- Staphiloccous aureus poisoning

b- Salmonella poisoning 19Gram - organism : klepseilla 20Yellow green discharge from vagina is : Trichomonus 21In the burn , role of : nine 22The best description of Case Study is : 23The best description of epidemolgy is : 24In Acute pancreaitis there is : a- Psedocust . b- Fistula ( my ejabah ) 25Typical case for epiglotitis 26The drug case optic neuritis is : Ethambutol 27Pain in breast spechaily above the aurola , most common cause is : a- Fibrocystic disease 28Pregnant devlope sudden left leg sweeling , best manegment is : a- Dopplex Rest Heparin 29Absoulute contraindication of lactation is : HIV 30When start lactation : as soon as possible 31Typical case for specific phobia 32Typical case for social phobia 33Agrophobia case 34Tretment of rash I forget the spell "diliak rash" : a- Typical steroid b- Typical antifungeal 35Baby with maculopapular rash : Rubela 36Rhumatoid fever "Typical " 37In blindeness what is the aerea that affected : Occiptal lobe 38In polycythemia cause of anemia is : Hypoviscosity 39In anemia of chronic disease there is : decrese iron and decrease TIBC 40SSRI 41Children eat the paper , what is the initial ttt : behivor ttt

42Case of ectopic pregnancy , site of pregnancy is : FT 43The abnormal sign in elbow X-Ray is : Posterior Pad sign 44Case " Ulnar compression " ttt : cubetus decompression 45Man with hand work by hummer came with pain on elbow diagnosis is : Lateral epicondlitit 46In pre-eclampsis the main sign is : elvated blood pressure 47Duration of drug in Rhumatoid fever is : a- 6 years b- 15 years 48Typical case of "Migrine" 49Typical case of "Uveitis " th 50degree of hemorride : heamoridectomy 51What is the food should avoid hyperlipedia patient : a- Avocado b- Organic meat 52What is the true for fracture of head and nech 53Regarding the CPR what is the true : a- 40 % recovery b- Provide 2 breath 54Patient he had multiple problem in his chest and he lives in crowded area what your action: a- Immunoglobulin b- H.influnza 55Patient with Rhumatoid arthritis on hand X-Ray there is swelling what you will do for him : a- NSAID b- Injection steroid 56Patient came to you with small swelling under his eye , on examination he have inflammation in lacrimal duct , you refer him to ophthalmologist before that what you will give him ? a- Topical steroid

bc57ab-

Topical antibiotic General antibiotic The best investigation for kidney function : 24 h collect urine Creatinine clearance www.3sam.cc

The End
" thanks a lot for all Doctors who have participating in writing of these collections Every 29th day of every month we will produce new edition inshallah Wish all the best for all Best wishes Abdullah Saleh AlHudaib

Вам также может понравиться